You are on page 1of 45

1. A 6-year-old girl is brought to your office 2.

A 6-year-old girl is brought to your office


with her parents. Her parents are worried with her parents. Her parents are worried
about her pubic hair and early puberty. about her pubic hair and early puberty.
She is health otherwise. On examination, She is health otherwise.
there are tanner stage 1 pubic hair On examination, there are tanner stage 1
(villous, absent pigmented or coarse pubic hair (villous, absent pigmented or
hair). There is no breast growth and her coarse hair). There is no breast growth
height is normal for her age. and her height is normal for her age.
Which ONE of the following is the most
appropriate next step in the examination? Which ONE of the following is the most
a. Follow up visit in 6 months appropriate next step in the examination?
b. Check serum FSH/LH
c. Bone age X-ray a. Follow up visit in 6 months
d. Pelvic CT scan b. Check serum FSH/LH
e. Vaginal examination with the help c. Bone age X-ray
of nurse d. Pelvic CT scan
e. Vaginal examination with the help
Answer detail: of nurse
The correct answer is a.
Answer detail:
This patient has developed isolated The correct answer is a.
growth of pubic hair before the age of 8
years. It is also called premature This patient has developed isolated
adrenarche. growth of pubic hair before the age of 8
years. It is also called premature
Usually there is no virilization and there adrenarche.
is no breast development. No
investigations ore required in most of Usually there is no virilization and there
the cases and only follow up is required is no breast development. No
in case child starts developing any investigations ore required in most of
features of congenital adrenal the cases and only follow up is required
hyperplasia. in case child starts developing any
features of congenital adrenal
Precocious puberty is characterized by hyperplasia.
appearance of following features before
the age of 8 years in girls: Precocious puberty is characterized by
appearance of following features before
- Development of secondary sexual the age of 8 years in girls:
characteristics.
- Accelerated growth velocity. - Development of secondary sexual
- Inappropriate sexual behavior. characteristics.
- Mood disorders. - Accelerated growth velocity.
- Inappropriate sexual behavior.
Investigations of suspected precocious - Mood Disorders.
puberty in a child include serum FSH,
LH, testosterone (males), estradiole Investigations of suspected precocious
(females). puberty in a child include serum FSH,
LH, testosterone (males), estradiole
Consider bone age x-ray (left wrist and (females).
elbow) and MRI if FSH and LH arc
increased. Consider bone age x-ray (oft wrist and
elbow) and MRI if FSH and LH are
increased.
Hepatomegaly is associated with
3. Which of the following is the indication for glycogen storage diseases and fatty acid
prostaglandin E-1 in a neonate? oxidation disorders. It can cause
hypoglycemia.
a. Transposition of great arteries
b. Hyperplastic left heart syndrome Micropenis shows congenital
c. Patent ductus aretriosus gonadotropin deficiency and possible
d. Renal failure pituitary abnormalities. This may result
e. Cardiac failure in cortisol deficiency. This may result in
hypoglycemia.
Answer detail:
The correct answer is a. Congenital pituitary deficiency can be
associated with midline defects such as
Prostaglandin (PGE-1) is indicated in cleft lip, cleft palate, single central
cardiac lesions that depend on patent incisor, and micro-ophthalmia.
cactus arteriosus to maintain adequate
pulmonary or systemic blood flow or to Hypoglycemia may be seen in these
promote adequate mixing. children as well due to pituitary failure.
Macrosomia occurs in infants of diabetic
These include: mothers and infants with severe
1-Pulmonary atresia with intact congenital hyperinsulinism.
ventricular septum.
2-Tricuspid atresia with intact Hypoglycemia is seen very commonly in
ventricular septum. neonates with a maternal history of
3-Critical coarctation of aorta. gestational diabetes and macrosomia.
4-Hypoplastic left heart syndrome.
5-Transposition of great arteries. 5. A 10-year-old male child is brought in by
his mother for ongoing exercised-induced
PGE-1 is riot indicated in hyper-plastic asthma despite being on maximum dose
left ventricular syndrome, patent ductus of inhaled corticosteroids.
arteriosus, renal failure and heart
failure. What would be next step in his
management?
4. You are working in a neonatal unit as an
intern. One of your patients developed a. Refer to respiratory physician
hypoglycemia, and you have given b. Add leukotriene receptor antagonist
2m1/kg of 10% dextrose. c. Cease inhaled steroids and start
intravenous steroids
Which one of the following is not d. Add long acting beta2 agonist
associated with neonatal hypoglycemia? e. Add short acting beta2 agonist

a. Hepatomegaly Answer detail:


b. Micropenis The correct answer is b.
c. Mid-line defects
d. Macrosomia Children with ongoing exercise-induced
e. Undescended testes asthma despite being on maximum dose
of inhaled steroids, should be given
Answer detail: leukotriene receptor antagonists as
The correct answer is e. these are superior and more effective
than long-acting beta-2 agonists.
Undescended testes are not associated
with syndromes causing hypoglycemia. Long acting beta-2 agonists induce
tolerance to short acting beta-2
agonists in children with asthma and 3- Pulmonary valve stenosis.
result in exacerbation of asthma 4- Webbed neck.
symptoms. 5- Failure to thrive, usually mild.
6- Abnormalities of cardiac conduction
Adding short-acting beta-2 agonist is and rhythm.
not correct as it is not preventer 7- Intellectual disability.
medication for asthma.
Myopia and ectopic ocular lens is
Intravenous steroids are required only consistent with Marfan syndrome and
in hospital settings for severe asthma these are not associated with Noonan
symptoms. Refer to respiratory syndrome.
physician is required if there is any
doubt in diagnosis or symptoms remain 7. A 2-year old girl presented with fever,
uncontrolled despite all step-up therapy neck stiffness, lethargy, and rhinorrhea
for last two days. She has a history of
6. A 2-year-old male child is brought to your meningococcal vaccine as part of routine
clinic with a 3 day history of fever, childhood immunization.
occasional cough and coryza. Upper
respiratory tract infection of viral On examination, she is febrile with
aetiology is suspected on clinical history. positive Kernig's sign.

On physical examination, patient is noted What is the most likely diagnosis?


having down-slanting of palpebral
fissures, widespread eyes, and low-set a. Bacterial meningitis
ears. Patient was also noted having short b. Viral meningitis
stature. c. Subarachnoid hemorrhage
d. Meningioma
All of the following are consistent with e. Atypical viral illness
this condition except?
Answer detail:
a. Pulmonary valve stenosis The correct answer is a.
b. Myopia and ectopic ocular lens
c. An autosomal disorder involving The most likely clinical diagnosis is
mutation in chromosome 11 meningitis. This child has all the clinical
d. Webbed neck features suggestive of bacterial
e. Abnormalities in cardiac conduction meningitis including fever, neck
and rhythm stiffness and positive kernig's sign.

Answer detail: The bacterial meningitis should be


The correct answer is b. assumed until proven otherwise and
emergency treatment should be
This clinical picture is suggestive of commenced as soon as possible without
Noonan syndrome. This is an autosomal waiting for investigations.
dominant disorder with mutation of
chromosome 11. It -as deep described The history of meningococcal vaccine
as a male Turner's syndrome but affects does not provide 100% immunity and
both sexes. Clinical features include the does not exclude the possibility of
following: bacterial meningitis.

1- Characteristic faces - down-slanting Meningococcal bacteria (Neisseria


palpebral fissures, widespread eyes, meningitides types A, B, C, Y and W135)
low-set ears + ptosis. are responsible for most meningococcal
2- Short stature diseases worldwide.
Most children outgrow laryngomalacia
Groups B and C are the most important by 18 to 20 months of age. Symptoms of
groups in Australia. There is vaccine laryngomalacia include:
available in Australia against group C
meningococcal bacterial but not against 1- Noisy breathing-an audible wheeze
group B meningococcal bacteria. when a baby inhale. It is aften worse
when the baby is agitated, feeding,
Although vaccine reduced incidence of crying or sleeping on his back.
meningococcal disease in the 2- High pitched sound
individuals who are vaccinated it does 3- Difficulty feeding.
not provide herd immunity. 4- Poor weight gain.
So this child can still have bacterial 5- Choking while feeding.
meningitis despite having been
vaccinated. Acute asthma makes breathing difficult
and wheeze is audible in any position
Also, it is mandatory to have baby adopts. The child usually looks
cerebrospinal fluid analysis to exclude unwell and it is uncommon before 6
bacterial meningitis. months of age

All other options are incorrect. Croup is acute inflammation of the


upper and lower respiratory tracts most
8. A mother brings her 2-week-old male commonly caused by parainfluenza
child with noisy breathing which gets virus type 1 infect on. It is characterized
worse when the baby is lying on his back. by a barking cough and inspiratory
The child is well otherwise. stridor. Croup affects mainly children
aged 6 months to 3 years.
What is the most likely cause of this
presentation? Bronchiolitis is an acute viral infection
of the lower respiratory tract affecting
a. Acute asthma infants less than 24 months and is
b. Laryngomalacia characterized by respiratory distress,
c. Croup wheezing, and crackles. It is primarily
d. Acute bronchiolitis caused by the respiratory syncytia virus.
e. Acute epiglottitis
Treatment is supportive with 02 and
Answer detail: hydration. Prognosis is generally
The correct answer is b. excellent, but some patients develop
apnea or respiratory failure. This baby
Laryngomalacia is a congenital does not have these symptoms.
softening of the tissues of the larynx
above the vocal cords. This is the most 9. What would be the most ominous sign in
common cause of noisy breathing in a child with croup?
infancy. The laryngeal structure is
malformed and floppy, causing the a. Degree of stridor
tissues to fall over the airway opening b. Increase in temperature
and partially block it. c. Subcostal retraction and use of
accessory muscles
Laryngomalacia symptoms are usually d. Loudness of stridor
present at bill, and can become more e. Low oxygen saturation
obvious within the first few weeks of
life. Answer detail:
The correct answer is c.
The croup is a viral illness and is after birth and late onset sepsis if it
characterized by following symptoms: happens after 48 hours of birth.

- Barking cough Risk factors for early onset sepsis


- Inspiratory stridor include:
- Widespread wheeze
- Work of breathing is increased - Maternal group B streptococcus
- Fever with no signs of toxicity colonization in current pregnancy.
- A previous baby with GBS infection.
The loudness of the stridor is not a good
guide to the severity of the obstruction. Risk factors for late-onset sepsis:
Hypoxemia is a late sign of significant
upper airways obstruction. - Artificial ventilation with an
endotracheal tube.
It is most common in 1 to 3-year-old - Extreme prematurity.
children and has the duration of 2 to 5 - Total parenteral nutrition.
days. Parainfluenza viruses are the
most common cause of croup, and Central catheter if present also becomes
antibiotics are not indicated. a reason for late-onset neonatal sepsis

The severity of croup is defined in 11.An 8-year-old boy is brought to the


research studies by the Westley score emergency department with photophobia,
but key features are: irritability and neck stiffness.

1-Mild airway obstruction: mild chest Lumbar puncture was performed and
wall retractions and tachycardia, but no cerebrospinal fluid analysis show 50
stridor at rest. neutrophils, 100 lymphocytes, normal
proteins and normal glucose.
2-Moderate airway obstruction: stridor
at rest, chest wall retractions, use of Which of the following is the most likely
accessory respiratory muscles and diagnosis?
tachycardia.
a. Tuberculosis meningitis
b. Bacterial meningitis
3-Severe airway obstruction: persistent c. Viral meningitis
stridor at rest, increasing fatigue, d. Normal CSF findings
markedly decreased air entry, marked e. Encephalitis
tachycardia.
Answer detail:
10.Which of the following is a risk factor for The correct answer is c.
early onset neonatal sepsis?
This patient has developed clinical
a. Artificial ventilation with an symptoms of meningitis and CSF
endotracheal tube analysis shows it to be most likely a
b. Previous baby with GBS infection viral meningitis.
c. Extreme prematurity
d. Total parenteral nutrition Please click on the image attached to
find out diagnostics of CSF analysis.
Answer detail:
The correct answer is b. 12.A 15-year-old child has recently been
diagnosed with type I diabetes mellitus.
Neonatal sepsis is categorized as either She is very motivated to learn about her
early onset defined as up to 48 hours
diet changes, to help her managing her 14.A mother brings her 3 weeks old son with
diabetes. history of excessive clear discharge from
both eyes and crusting over the
What is the most appropriate advice? eyelashes. There is no redness or
irritation around the eyes.
a. Avoid high protein diet What is the most likely diagnosis?
b. Avoid low carbohydrate diet
c. Take foods with high glycemic index a. Conjunctivitis
d. Increase intake of saturated fats b. Nasolacrimal duct obstruction
e. Avoid mono-unsaturated fats c. Glaucoma
d. Allergic reaction
Answer detail: e. Meningitis
The correct answer is a.
Answer detail:
The patients with type 1 diabetes The correct answer is b.
mellitus should avoid high protein and
should consider low carbohydrate diet. This neonate has symptoms of blocked
or narrow nasolacrimal duct.
She should consider reducing intake of
saturated fats and substitution of Clear eye discharge and occasionally
saturated fats with monounsaturated crusting over the eyelashes without
and polyunsaturated fats. redness or irritation are commonly seen
in newborn and infants and result from
13.A mother of one week old neonate is on narrow or obstructed nasolacrimal
methadone programed. She is concerned ducts.
about the risk of methadone to her baby
as she plans to breast-feed her baby. This condition usually resolves
Which of the following is the most spontaneously and requires antibiotics
appropriate advice? only when complicated by infection
(conjunctivitis or dacryocystitis).
a. Breast feeding is contraindicated
b. She should breastfeed the baby Ophthalmologic consultation for
before taking methadone dose nasolacrimal duct probing is appropriate
c. She should use formula milk mixed if this problem persists past 12 months
with breast milk of age or earlier if complicated by
d. Leave methadone program for four recurrent infection.
months and then rejoin
e. Continue methadone program and 15.Bronchiolitis is a viral illness which results
continue breastfeeding in a variety of respiratory symptoms in
children younger than two years of age.
Answer detail:
The correct answer is e. Which one of the following is considered
to be at high risk of airway compromise if
Various studies have shown that the developed bronchiolitis?
amount of methadone transferred
through milk is low even at higher doses a. Infants younger than three months
of methadone. of age
So it is unlikely to result in toxicity to b. Term babies
infant. Also it may help to lessen the c. Maternal smoking
symptoms of neonatal abstinence if d. First degree relative with asthma
mother was on methadone before e. Another sibling with history of
delivery of the baby. bronchiolitis in past 12 months
Answer detail: history is inconsistent with the
The correct answer is a. presentation.

Bronchiolitis is a viral illness caused by There are rare causes of the rib fracture
a respiratory syncytial virus. in children which include
cardiopulmonary resuscitation, birth
In the presence of following risk injuries and osteogenesis imperfect.
factors, the risk of airway compromise
goes higher and thus these children The pneumonia is never a cause of rib
should be admitted to the hospital if fractures.
they develop bronchiolitis:
The case needs to be notified of the
- Infants under three months of age child protection services. The
(risk of apnea). notification is a mandatory and legal
- Preterm or low birth weight infants requirement.
- Infants with chronic lung disease.
- Infants with congenital acyanotic 17.An 11-year-old male child is brought to
heart disease. emergency following a fall into an unfilled
swimming pool and hit his head on the
16.A 6-months-old male child is brought to concrete surface. Vital signs include heart
the emergency department by his rate of 86/min, respiratory rate of 19/min
parents, for assessment of his breathing and is hypotensive (50/30 mmHg)
difficulty and irritability. On examination, despite adequate fluid resuscitation. CT
the temperature is 38.3c, heart rate is scan of the head is normal. Chest
140 beats per minute, oxygen saturation examination is clear.
is 97% on room air and respiratory rate is
24 breaths per minute. What is most likely diagnosis?

There are bruises on the chest wall and a. Neurogenic shock


he is using accessory muscles. The chest b. Hypovolemic shock
x-ray is done which shows posterior rib c. Obstructive shock
fractures and right middle lobe d. Cardiogenic shock
pneumonia. The treatment of pneumonia e. Inadequate fluid resuscitation
is carried out as per local protocol.
Answer detail:
What is the most common cause of this The correct answer is a.
type rib fractures?
This patient is most likely suffering from
a. Non-accidental injury neurogenic shock. Loss of sympathetic
b. Cardiopulmonary resuscitation tone prevents the expected tachycardia
c. Birth injuries response.
d. Osteogenesis imperfect
e. Pneumonia The hallmarks of neurogenic shock are
hypotension with either bradycardia or
Answer detail: normal heart rate despite fluid
The correct answer is a. resuscitation.

The commonest cause of the posterior If the hypotension cannot be corrected


rib fractures is a non-accidental injury. with fluids only, vasopressor therapy
may be required.
There is usually no history of trauma
under these circumstances or the As chest examination is clear, it is
unlikely obstructive shock (cardiac
tamponade, pneumothorax). Also it is The correct answer is e.
unlikely to be primary cardiogenic or
hypovolemic shock. This child has signs and symptoms
consistent with diagnosis of Giardiasis.
Inadequate resuscitation is not a It is caused by a parasite called as
correct option as information provided Giardia lamblia.
indicated adequate fluid given.
Symptoms of Giardiasis include:
18.A 6-year-old male Aboriginal child has
ingested 2 lithium batteries 2 hours ago. 1. Diarrhea, which may he watery,
Chest and abdominal X-rays confirmed usually lasting more than 1-2 weeks.
the position of the batteries in 2. Weight loss.
esophagus. His Parents are quite anxious. 3. Frequent loose or pale, greasy stools
which may float in the toilet bowl. 4.
What will you consider next? Fatigue.

a. Emergency endoscopic removal Lactose intolerance may develop in 20-


b. Watch for the passage for 3 days 40% of cases of giardiasis and lost
c. Serial chest and abdominal x-rays severer weeks. However it is not
d. Urgent laparotomy primary diagnosis here.
e. Reassure the parents
Coeliac disease presents after
Answer detail: introduction of glutin-containing solid
The correct answer is a. diet in a child during weaning. It starts
during first year of life.
If a child is ingested lithium batteries
and if these are 'n esophagus, it Cystic fibrosis presents with frequent
requires emergency endoscopic removal respiratory and gastrointestinal
within 6 hours (as soon as possible). infections from the birth. This child was
previously healthy, so cystic fibrosis is
Otherwise electrical current generate by unlikely diagnosis.
these batteries can lead to destruction
of mucus membrane and perforation of 20.A mother calls you on phone about her 8-
the esophagus in less than 6 hours. year-old son with type 1 diabetes
mellitus. Child is not feeling well and is
All other options are incorrect. still conscious. His blood sugar is 2.1
mmol/L.
19.A mother brings her 3 year-old male
Aboriginal child for passing loose stools What is the immediate next step?
over last 2 weeks. The child lost 2 kg
weight after the onset of diarrhea. Child a. Give him insulin
looks fatigued. Clinical examination is b. High carbohydrate diet
remarkable for mild dehydration. c. Give him glucagon
d. Call ambulance and reach
What is the most likely diagnosis? emergency department as soon as
possible
a. Lactose intolerance e. Reassure her that things are still in
b. Food poisoning control
c. Coeliac disease
d. Cystic fibrosis Answer detail:
e. Giardiasis The correct answer is b.

Answer detail:
Young children with type 1 diabetes are In the past, Aboriginal and Torres Strait
particularly at risk for severe Islander people have suffered from
hypoglycaemia because their exercise much higher rates of tuberculosis, more
and food intake are not predictable, than 20 times the rate of non-
they have increased sensitivity to indigenous Australian born people in
insulin, and it may not be possible to some areas.
make sufficiently small adjustments to
their insulin regimen. While there have been substantial
improvements in disease rates in recent
If the patient is conscious and able to decades, tuberculosis remains more
drink and swallow safely, a rapidly- common in Indigenous than non-
absorbed carbohydrate (glucose tablets, Indigenous Australian born people in
glucose gel, table sugar, fruit juice, or many parts of Australia, particularly
honey) should be given by mouth. northern and central Australia.
This process may be repealed in 10 to
15 minutes. Although there is uncertainty about the
efficacy of BCG in preventing pulmonary
Infants and children with altered tuberculosis, it provides substantial
consciousness and inability to swallow protection against disseminated forms
rapidly absorbed carbohydrate safely, of the disease in young children is not
should be treated with intravenous effective in complete prevention of
dextrose. pulmonary tuberculosis.
While placing the intravenous lines,
subcutaneous or intramuscular BCG is therefore recommended for
glucagon should be considered. Aboriginal and Torres Strait Islander in
regions of high incidence.
21.A mother of an Aboriginal neonate visited
you for advice about the BCG vaccine and It is usually administered to eligible
tuberculosis. infants by hospital staff (midwives or
nurses who have been specially trained)
Which of the following is incorrect with soon after delivery.
regard to the BCG vaccine and
tuberculosis among Aboriginal and Torres Injection technique is particularly
Strait Islander people? important for BCG vaccination which
must be administered intradermally.
a. Aboriginal and Torres Strait
Islander people suffered from Adverse events, such as regional
tuberculosis 20 times the rate of non- lymphadenitis, are less common when
Indigenous Australian-born people administration is performed with proper
b. BCG vaccine is found to be effective technique.
in preventing pulmonary tuberculosis
c. BCG vaccine protects against 22.A 6-year-old girl is accompanied by her
disseminated tuberculosis in young parents. They are worried about her as
children she started developing her pubic hair,
d. It is usually administered to eligible breasts and displays inappropriate sexual
infants by trained staff behavior.
e. BCG vaccination is administered
intradermal On examination, She height is more for
her age and she has breast and pubic
Answer detail: hair at Tanner stage 3.
The correct answer is b.
What will do next?
a. Reassure undergo a hip ultrasound and referral to
b. Follow up in 6 month an orthopedic surgeon.
c. Serum FSH, LH
d. MRI brain During first six months of age, the
e. Monitor growth velocity ultrasound of the hip is the
investigation of choice.
Answer detail:
The correct answer is c. 24.An 18-year-old girl presented with
delayed puberty. She is at Tanner stage 2
This girl has diagnosis of precocious now. Which of the following is the initial
puberty and needs investigations to find investigation of choice in this situation?
out the cause.
Precocious puberty is characterized by a. Chromosomal analysis
appearance of following features before b. Serum FSH and LH level
the age of 8 years in girls: c. Thyroid function
d. ESR
- Development of secondary sexual e. Bone age
characteristics.
- Accelerated growth velocity. Answer detail:
- Inappropriate sexual behavior. The correct answer is e.
- Mood disorders.
This patient presented with delayed
Investigations of suspected precocious puberty which is arrested at Tanner
puberty it this girl, include serum FSH, stage 2. Tanner stage 2 is usually seen
LH and estradiol (females). in girls between 9 to 13 years of age
and Tanner stage 5 is seen at 16 years
Consider bone age x-ray (left wrist and of age. There is 5 year pubertal delay in
elbow) one MRI if FSH and LH are this patient and needs to be
increased. investigated.

23.Which of the following is the best The most common cause of pubertal
screening test for developmental delay is constitutional delay. The initial
dysplasia of the hip at 8 months of age? choice of investigation is to determine
a. Ultrasound bone age by doing an X-ray of left wrist
b. CT and left hand.
c. MRI
d. X-ray Bone age determines the biological age
e. Abnormal hip examination instead of actual age. It helps to
determine the remaining growth
Answer detail: potential for the adolescent. Bone age is
The correct answer is d. useful in conjunction with chronological
age and height/weight.
The x-ray of the hip is the invest gallon
of choice for screening developmental Bone age is usually delayed in
dysplasia of the hip in beyond six constitutional delayed puberty and is
months of age. normal in Turner's syndrome. Delayed
puberty is defined as lack of pubertal
All babies with risk factors for development by 14 years of age for girls
developmental dysplasia of the nip and 15 years for boys.
(Breech presentation, Oligohydramnios,
Family history of DDH and Underlying As this patient has started pubertal
neuromuscular dysfunction) or with an development and it is arrested at
abnormal examination of the hip should
Tanner stage 2, it is unlikely due to available for GBS and this child world
hormonal disorder. have missed that.
However if bone age determination is
not helpful, an endocrinologist may Sill it is worth mentioning that incidence
consider doing serum FSH, LH, prolactin, of meningitis in Aboriginal infants is
thyroid function tests and chromosomal about 3% higher than rest of the
analysis. Australian population.
Similarly malnutrition and post maturity
25.A 2-week-old Aboriginal child presents has no proven association with
with fever. On examination, you find increased incidence of neonatal
bulging fontanelle. On cerebrospinal fluid meningitis.
culture, streptococcus organism was
isolated. 26.A 13 year-old boy who is currently in a
special school is now at verge of
Which of the following is the cause of this expulsion due to sudden outbursts of
presentation? anger in school and at home over a
period of last three months. What would
a. Aboriginal background be the most appropriate drug
b. Malnutrition management for this situation?
c. Prematurity
d. Postmaturity a. Sodium valproate
e. IgA deficiency b. Lithium
c. Haloperidol
Answer detail: d. Risperidone
The correct answer is c. e. Olanzapine

Group B Streptococcus is the most Answer detail:


common cause of life threatening The correct answer is d.
infections in newborn infants. Babies
may be exposed during birth to Group B This child is a special child and most
Streptococcus found normally it the likely suffers from autism considering
vagina. his behavior and requirement of special
school. Aggression and irritability can
Babies are at increased risk of be a feature of autism especially during
contracting meningitis or septicaemia if adolescence. Atypical antipsychotics
have been seen to improve behavioral
(a) They are low birth weight. symptoms such as repetitive behavior,
(b) Premature. hyperactivity, irritability and
(c) Born after prolonged labor. aggression. Risperidone, an atypical
(d) Premature rupture of membrane. antipsychotic is the drug of choice for
management of anger outbursts in
Currently there is no vaccine for Group children with autism in Australia and is
B streptococcus. approved under patient benefit scheme
(PBS). Typical antipsychotics like
IgA deficiency causes increased haloperidol can cause extra-pyramidal
frequency bronchitis, chronic diarrhea, symptoms and restlessness which can
conjunctivitis and otitis media in further exaggerate the symptoms in
children. It cues not increases incidence children with autism. Long-term use can
of meningitis. result in tardive dyskinesia. Olanzapine
is not considered as first line drug for
Aboriginal background would have been management of aggressive behavior in
an option if there was any vaccine children with autism due to side-effects
like weight gain and development of
type 2 diabetes mellitus. Lithium and stasis associated with underlying
sodium valproate both are mood anatomical abnormalities including
stabilizers and should be used in bipolar strictures, jejunal diverticulosis, motility
disorder. This patient does not have disorders and various operations that
signs or symptoms of bipolar disorder. create blind loops.

27.A 20-month-old baby is brought in by his Clinical manifestations of bacterial


parents with chronic diarrhea and weight growth include diarrhea, weight loss
loss. On examination, the baby has and anaemia due to vitamin B12
growth retardation, muscle wasting and deficiency.
distended abdomen with hyperactive
bowel sounds. Chest examination is clear. 28.A mother brings her 12-month old child
Laboratory investigations show decrease with history of loose watery stools for the
serum iron or folate levels. Patient has past few hours.
been on the lactose-free diet for 3 days
without resolution of symptoms. Which of the following is the most specific
indicator of severity of dehydration in this
What is the most likely diagnosis? age group?

a. Primary lactose intolerance a. Sunken eyes


b. Coeliac disease b. Irritability and lethargy
c. Cystic fibrosis c. Increased respiratory rate
d. Bacterial over growth d. Urine output
e. Chronic liver disease e. Dry mucous membrane

Answer detail: Answer detail:


The correct answer is b. The correct answer is c.
This patient has the clinical diagnosis of
coeliac disease. It presents as severe Specific indicators of severity of
malabsorption syndrome in children dehydration include:
with chronic diarrhea, failure to thrive,
muscle wasting and hyperactive bowel - Percentage of weight loss.
sounds. - Prolonged capillary refill time.
- Abnormal skin turgor
Primary lactose tolerance is due to - Increased respiratory rate.
deficiency of lactase in a small intestine
and is a malabsorpton syndrome Following signs considered for
presenting with abdominal rain, assessment of dehydration however
boating, distension, flatulence and these are less reliable:
watery diarrhea. It usually resolves
with a lactose-free diet foe about 2 - Sunken eves.
weeks. - Lethargy
- Dry mucous membranes.
Cystic fibrosis is an inherited autosomal - Irritability and lethargy.
recessive d souse and is trio most
common cause of bronchiectasis and Urine output is measured to monitor the
chronic suppurative lung disease in response to fluid therapy in
Caucasian children. dehydration.

Bacterial overgrowth in the small bowel 29.A mother brings her 3-year old child with
causes diarrhea and nutrient history of uncomfortable defecation and
malabsorption. Most patients with minimal bright bleeding on toilet paper
bacterial overgrowth have intestinal after defecation. On rectal examination
with mother's consent, hard stools and - Athetoid cerebral palsy with or
painful anal fissure noticed. without seizures.
- Developmental delay.
What is the immediate next step? - Hearing deficit.
- Oculomotor disturbances including
a. Laxatives and high fiber diet paralysis of upward gaze.
b. Reassurance as it is normal variant - Dental dysplasia.
c. Apply anusol cream on the fissure - Intellectual impairment.
d. Abdominal X-ray
e. Abdominal ultrasound Kernicterus does not cause
hypothyroidism in long term.
Answer detail:
The correct answer is c. 31.Which one of the following can be an
alarming sign in a 2 month old infant?
Anal fissures are often seen in infants
and toddlers with uncomfortable a. Inability to smile at people
defecation and minimal bright red b. Inability to hold neck
bleeding. The anal mucosa is split in the c. Inability to explore things by
midline either anteriorly or posteriorly. holding, feeling and looking at them in
It is caused by passage of hard stools. their hands and putting them in their
The fissure usually heals within few mouths
days after application of topical anusol d. Does not appear to recognize their
cream. mother
Topical anusol cream is effective in e. Does not appear interested in
healing anal fissures in children. It activities around them
contains zinc oxide, benzyl benzoate
and balsam. Once the pain for anal Answer detail:
fissure is under control) advise using The correct answer is a.
laxatives and fiber diet.
No further investigations needed. It is an alarming sign when a 2-months
old child is unable to do following
30.A one week old neonate developed severe things:
jaundice and mother is quite concerned
about the long term risk of severe 1. Does not respond to loud sounds.
hyperbilirubinemia. 2. Does not watch things as they move.
3. Does not smile at people
Which of the following is not a possible 4. Does not bring hands to mouth.
consequence of kernicterus in this baby? 5. Cannot hold head up when pushing
up when or tummy.
a. Hearing deficit
b. Athetoid cerebral palsy So if a child at 2 months of age does not
c. Paralysis of upward gaze smile at people-it is a concerning thing
d. Hypothyroidism for a mom and needs thorough
e. Dental dysplasia evaluation.

Answer detail: Neck holding is achieved by 3-months.


The correct answer is d.
Ability to explore things by holding,
Kernicterus is the pathogenic sign or feeling and looking at them in their
bilirubin staining of the brain stem hands and putting them in their mouths
nuclei and cerebellum. is achieved by 6 months of age.
Clinical findings include:
A skill to recognize mother's face is
achieved by 9 months of age. Intraluminal esophageal electrical
impedance has not been thoroughly
If a child by 9 months of age, does not validated and normal values have not
appear interested in activities around been determined in the pediatric age
them, it is abnormal and requires group.
further evaluation.
Starting treatment is an option after
32.An 18-month-old baby is brought by her confirmation of diagnosis.
mother with a long history of vomiting
after every feed. The baby has a poor 33.A mother brings her 4-week-old child who
appetite and does not eat much. Her vomits after each feed. The child is
weight is at the 40th percentile. The rest lethargic and hungry. On examination, no
of the physical examination is normal. epigastria mass is found.

What will be the next step in Which of the following investigation will
management? confirm the diagnosis?

a. Intraesophageal pH probe a. Barium meal


monitoring b. Ultrasound
b. Reassurance c. Urine analysis
c. Esophagogastroduodenoscopy d. Abdominal X-ray
d. Intraluminal esophageal electrical e. Stool culture
impedance
e. Start treatment and give a trial of Answer detail:
proton pump inhibitor The correct answer is b.

Answer detail: This child has provisional diagnosis of


The correct answer is a. hypertrophic pyloric stenosis. the
investigation of choice in this situation
This child has a clinical diagnosis of is abdominal ultrasound.
gastro esophageal reflux disease.
Vomiting could be cue to motility Hypertrophic pyloric stenosis is more
disorder as well. common among males by a 4:1 ratio,
particularly first born males. It occurs
So intraesophageal monitoring of pH is most often between 3 to 5 weeks of age
considered the next best step in and rarely after 12 weeks.
management of such cases to confirm
the diagnosis. Symptoms car develop between 2 sic 6
weeks of life. Projectile vomiting
Reassurance is not enough as baby has (without bile) occurs shortly after
lost weight and this requires further eating. Until dehydration sets in. the
assessment. child feeds avidly and otherwise
appears well, unlike many of those with
Esophagogastroduodenoscopy allows vomiting caused by systemic illness.
visualization of the mucosa for
diagnosis of peptic ulcer disease. It also Gastric peristaltic waves may be visible,
helps to find out H. pylori infection, crossing the epigastrium from left to
strictures, and peptic esophagitis. It right. A discrete, 2- to 3-cm, firm,
provides access to obtain biopsies for movable, and olive-like pyloric mass is
histopathology examination. However, sometimes palpable deep in the right
this is not the required for diagnosis of side of the epigastrium.
gastro-esophageal reflux disease.
However these classic findings are not g) o large for gestational age (more
present in every case and diagnosis than 90th percentile).
should still be suspected based on
history. Indication to check fetal scalp pH is to
assess degree of fetal hypoxia during
Diagnosis is confirmed by abdominal fetal distress. ECG, CT head and full
ultrasonography showing increased blood count are not the primarily
thickness of the pylorus along with an required investigations in this baby.
elongated pylorus. The classic
electrolyte pattern of an infant with 35.A 2-week-old neonate is found to have
pyloric stenosis is that of hypochloremic discharge and redness in both eyes. On
metabolic alkalosis. examination, there is mild hyperemia
with a thick mucopurulent discharge.
34.A newborn baby of a diabetic mother was
born with 4500 g with normal vaginal What is the most likely diagnosis?
delivery without any complication. APGAR
score at one minute was 5 and at 5 a. Chemical conjunctivitis
minutes, APGAR score 7.Peripheral b. Chlamydia conjunctivitis
cyanosis was present. After few hours, c. Gonococci conjunctivitis
the baby was found to be jittery. d. Meningitis
e. Viral infection
What is the investigation of choice?
Answer detail:
a. Scalp pH The correct answer is b.
b. CT head
c. Blood sugar level Chemical conjunctivitis due to ocular
d. Electrocardiography prophylaxis usually occurs on the first
e. Full blood count day of life. Gonococci conjunctivitis
generally has its peak time of onset
Answer detail: between 3 and 7 days after birth.
The correct answer is c.
By the end of the first week of life and
The 5 minute APGAR score is regarded throughout the first month of life,
as more important and reflective of the Chlamydia becomes the most frequent
baby's overall health, than the 1 minute cause of conjunctivitis.
APGAR score.
So at 2 weeks of life, this neonate has
A total APGAR score of B-10 is regarded developed signs and symptoms of
as good, 5-1 can indicate that the baby chlamydia conjunctivitis.
is mildly unwell, 3-4 reflects moderate
poor health and 0-2 is very poor health. Chlamydial conjunctivitis can vary in
severity, ranging from mild to severe
This baby has developed hypoglycemia hyperemia with a thick mucopurulent
most likely considering the history of discharge and pseudo membrane
maternal diabetes and macrosomia. formation.

Maternal hyperglycemia can result in Gonococci conjunctivitis usually


fetal hyperglycemia and then secondary presents at 2 to I days of life with
fetal hyperinsulinism. Insulin is the intense bilateral bulbar conjunctiva
main 'growth hormone' of the fetus and erythema, chemosis, and a copious
therefore infants of diabetic mothers purulent discharge.
are often macrosomic (more than 4000
All infants with gonococci conjunctivitis Her parents were advised by her teachers
require admission and evaluation for to seek medical help. When you talk to
disseminated disease. the girl, she becomes tearful and looks
Intravenous third generation withdrawn.
cephalosporin antibiotics are first lire of
management. What do you suspect from this story?

36.A 6-month-old male infant is brought to a. Separation anxiety


the hospital by his parents. He has been b. Truancy
having bilious vomiting, cramp abdominal c. Depression
pain, mild abdominal distension and d. Conduct disorder
passing blood and mucus in the stool e. ADHD
over last 3 hours.
Answer detail:
What is the most appropriate diagnosis? The correct answer is b.

a. Intestinal malrotation with midgut This child has clinical features


volvulus suggestive of diagnosis of truancy.
b. Congenital hypertrophic pyloric Escaping the school and going to games
stenosis and parlors on one hand and being
c. Urinary tract infection tearful and withdrawn on other hand
d. Small intestinal atresia does not fit into diagnosis of
e. Raised intracranial pressure depression.

Answer detail: Truancy is any intentional unauthorized


The correct answer is a. or illegal absence from compulsory
schooling. Children who run away from
This clinical diagnosis in this situation is school to do other things (truancy)
intestinal malrotation with volvulus. rather than stay home usually have
different problems from those who don't
Infants often present acutely with want to go to school.
midgut volvulus, manifested by bilious They may be wanting to get attention,
vomiting, cramp abdominal pain, trying to impress their friends or they
abdominal distention, and the passage may be angry because of school or
of blood and mucus in their stool. home problems. Truancy may happen
when there are learning problems.
Patients with chronic, uncorrected Children who truant a lot sometimes go
malrotation can have recurrent on to break the law as they get older.
abdominal pain and vomiting.
Malrotation can also be entirely Children who truant usually try not to
asymptomatic. let their parents find out truancy
sometimes happens when parents are
With acutely ill patients, consider not very interested in the child getting a
emergency surgery laparotomy if there good education, and perhaps don't get
is a high index of suspicion. So urgent on very well with the school
surgical referral should be considered to themselves.
prevent bowel gangrene.
Schools usually expect parents to
All other options are incorrect. inform the school in advance if their
child will be absent through the student
37.A mother brings her 8-year old daughter diary, or by phoning the school on the
with history of truancy. The girl avoids day of absence. It is a good way of
school and goes to games and parlors. ensuring child's safety.
Any infant who contracts hepatitis B
After 3 days of unexplained absence the during this period has more than 90%
school will usually contact the parent, chance developing chronic hepatitis B
as schools are required to report infection with increasing the risk of
student absences. hepatocellular carcinoma latter in the
life.
Children with depressive disorders feel
sad, lack interest in activities they 39.A 12-year-old child has history of
previously enjoyed, criticize recurrent urinary tract infections since
themselves„ and are pessimistic or the age of 3 years. Which ONE of the
hopeless about the future. Thinking that following is the investigation of choice to
life is not worth living, they may rule out renal scarring?
contemplate suicide. They may also be
irritable ant aggressive. They may a. Renal biopsy
indecisive, and lave problems b. Renal ultrasound
concentrating. They tend to lack energy c. Serum urea and creatinine
and to have problems sleeping. d. DMSA scan
e. CT scan of abdomen
The typical behavior of those with
conduct Disorder involves bullying, Answer detail:
frequent physical fights, Deliberate The correct answer is d.
destruction of other people’s property.
Breaking into houses or cars, staying Recurrent urinary tract infection (more
out late at night despite parental than two times during childhood) can
prohibition, running away from home, lead to renal scarring, hypertension, and
or frequent truancy from school. end-stage renal dysfunction.
Vesicoureteral reflux is the usual
This child does not meet the criteria for etiology leading to these complications.
conduct disorder, ADHD, depression and
separation anxiety. Vesicoureteral reflux (VUR) is the
retrograde passage of urine from the
38.What is the risk for prenatal viral bladder into the upper urinary tract.
transmission in infants born to a mother
who is positive for both HbsAg and The dirnercaptosuccinic acid
HBeAg? scintography scan (DMSA) is the gold
standard for diagnosis of kidney
a. 85% scarring and measurement of
b. 25% differential renal functions.
c. 5%
d. 3% Ultrasound is generally requested after
e. No risk of transmission present first urinary tract infection to rule out
any anatomic abnormality. It is not the
Answer detail: gold standard investigation to diagnose
The correct answer is a. kidney scarring.

If a mother is HBsAg and HBeAg 40.A 14-year-old boy is brought to the


positive, the risk for transmission of emergency department with fever and
hepatitis B to baby is 70 to 90%. painful, tender and warm knee joint. He
would not move his limb and does not
The risk is lowered significantly to 8 to allow anyone to touch it.
15% if mother is HBsAg positive and
HBeAg negative. a. Osteomyelitis
b. Septic arthritis
c. Perthe's disease The correct answer is b.
d. Traumatic injury
e. Osteonecrosis This child has developed clinical
features of Kawasaki disease. It is self-
Answer detail: limited vacuities of unknown etiology.
The correct answer is b. Around 80% cases occur before the age
of 5 years.
This child has a clinical diagnosis of
septic arthritis. Septic arthritis is The diagnosis of Kawasaki should be
characterized by following main considered in any child who continues
features: to have fever beyond five days. 1 he
other features include bilateral
- Fever conjunctivitis, oral mucosal ulcers,
- Acute onset of limp or refusal to use cervical lymphadenopathy, redness of
the limb. hands and feet and maculopapular
- Pain on movement and al rest. generalized rash.
- Limited range or loss of movement.
- Soft tissue redness and swelling often The other differential diagnosis is also a
present. possibility however persistent fever
beyond 5 days along with skin changes
The child with osteomyelitis the lower in peripheries suggest Kawasaki disease
extremity is characterized by following as top diagnosis here.
features:
Kawasaki disease is associated with
- Sub-acute onset of limp vacuities which can lead to coronary
- Localized pain and pain on movement. artery aneurysm.
- Tenderness.
- Soft tissue redness and swelling may 42.A 6-year-old child has been brought to
be absent. you by parent for nighttime bed-wetting.
-Fever may be absent. He has been hygiene trained and has
been dry during daytime from the age of
Referral to orthopedic surgeon is 4 years.
required based on clinical diagnosis of
septic arthritis or osteomyelitis in Which of the following you are most likely
children. to find in the history?

41.A 4-year-old child is being seen in your a. Night terrors


office with a history of 5 days of ongoing b. History of urinary tract infections
fever and rash. On examination, his temp c. History of urinary tract infection in
is 38.2. Also, there are red injected mother while pregnant
conjunctiva red cracked lips, d. Positive family history of similar
maculopapular rash, redness of palms complaints
and soles and cervical lymphadenopathy. e. Stress

What is the most likely diagnosis? Answer detail:


The correct answer is d.
a. Scarlet fever
b. Kawasaki disease Primary enuresis is defined as daytime
c. Respiratory tract infection bed wetting after the age or 4 years or
d. Tonsillitis night time bedwetting after the age of 6
e. Non-specific viral illness years. It appears to be due to delayed
maturation of achieving bladder
Answer detail: continence.
well with the percent predicted value
This child has been dry during daytime for the forced expiratory volume in one
and thus on y suffers from nocturnal second (FEV1).
enuresis.
Peak flow meters have a limited role in
Positive family history with similar establishing the diagnosis of asthma in
complaints is the most important risk the office. However, they are very
factor as the condition runs in the useful in measuring the severity of
families. asthma exacerbation.
Also nocturnal enuresis is more common
in boys. Peak flow readings are higher when
patients are well, and lower when the
Secondary enuresis is wetting after airways are constricted
normal continence of at least 3 months.
It may be caused by psychological Seventy of wheeze is not reliable sign of
stress-like separation from parents or severity of asthma. Severe asthma
arrival of new baby. usually presents with silent chest and
low grade wheeze.
Control of urine is usually expected by
the age of 5 in majority of the children. Response to inhalers and degree of
dyspnea are also very unreliable signs
After the age of 6, invest gallons to assess the severity of asthma. A
including intravenous program or patient with mild asthma may be
ultrasound are necessary to exclude equally breathless as with severe
urinary tract abnormality. asthma.

43.A 12-year-old male child is brought in Pulmonary function testing is done at


hospital by his mother with acute specialized centers and is required to
shortness of breath and wheeze. He has diagnose asthma. However peak flow
background history of asthma and uses measurement is quick and useful in
inhalers as required. emergency situation to assess the
severity of asthma.
Which of the following is the best bed-
side measure to assess the severity of 44.A 4-year-old male child is brought to the
asthma? emergency department by an ambulance
with 3 days history of fever, associated
a. Severity of wheezing altered consciousness. There has been
b. Response to inhalers progressive personality changes and
c. Degree of dyspnea occasional drowsiness.
d. Peak expiratory flow rate
e. Pulmonary function tests An emergency lumbar puncture is done
which confirms the diagnosis of viral
Answer detail: meningitis.
The correct answer is d.
Which of the following is the best choice
The peak expiratory flow rate (PEFR, for this condition?
also known as a peak flow) is the
maximal rate that a person can exhale a. Ceftriaxone intravenously (IV)
during a short maximal expiratory effort b. Benzyl penicillin IV
after a full inspiration. c. Ciprofloxacin IV
d. Acyclovir IV
In patients with asthma, the PEFR e. Cefuroxime IV
percent predicted correlates reasonably
Answer detail: Most of the children grow out of asthma
The correct answer is d. when they become adults and very few
continue to be adult asthmatics.
This case is suggestive of viral
encephalitis. Encephalitis is an Giving 6 to 10 inhaled doses of beta-2
inflammation of brain parenchyma. It is agonist are very safe and appropriate if
mainly caused by viruses although other used in severe asthma.
organism’s including bacteria,
mycoplasma, rickettsia and Histoplasma 46.What is the treatment for a child with an
can cause encephalitis. altered level of consciousness who is
found to have hypoglycemia?
Suspect it when a viral prodrome is
followed by irrational behavior, altered a. 2ml/kg of 50% dextrose IV
consciousness and possibly cranial b. 5ml/kg of 10% dextrose IV
nerve lesions. c. 2ml/kg of 10% dextrose IV
d. 1ml/kg of 25% dextrose IV
In this case, the cerebrospinal fluid e. 2ml/kg of 4% dextrose IV
analysis reveals viral meningitis thus
viral etiology is entertained. Answer detail:
The correct answer is c.
This patient should he hospitalized and
treatment will be supportive Suspected Any child who presents with altered
herpes simplex encephalitis should be level of consciousness must have blood
treated with intravenous acyclovir glucose tested in the emergency
immediately. department.

45.Which of the following statement is true The current treatment of hypoglycemia


regarding asthma in children? in the child is 2 ml/kg 10% dextrose.

a. Inhaled bronchodilators are very 47.A 12-year-old male child is brought into
effective under age of 12 months emergency department with loss of
b. Oral bronchodilators are very consciousness and low blood glucose level
effective under age of 12 months is 2.5 mmol/L. He has past medical
c. Use a spacer with face mask for history of type 1 diabetes mellitus and is
children age between 1 to 2 years on insulin.
d. Most of the children do not grow
out of asthma when they become What is the most appropriate
adults e. Giving more than 6 inhaled management?
doses of beta-2 agonist can be life
threatening in severe asthma a. 10% glucose intravenously
b. 5% glucose intravenously
Answer detail: c. 50% glucose intravenously
The correct answer is c. d. Normal saline
e. Oral sweet juice
Bronchodilators inhaled or oral, both are
ineffective under 12 months. Answer detail:
The correct answer is a.
Use spacer with face mask in children
age between 1-2 years for adequate In children with type 1 diabetes
delivery of medication. mellitus, hypoglycemia can lead to
seizures and loss of consciousness.
The most appropriate treatment of
severe hypoglycemia (associated with 49.A 4-week old infant was brought to your
seizures, loss of consciousness) in clinic for well-baby checkup. The baby is
hospital settings include 10% glucose feeding well and gaining weight, but the
intravenously 2 ml/Kg bolus followed by mother said she notices a bulge in his
sodium chloride 0.45% with glucose 5% inguinal area every time he cries. You
intravenously as maintenance fluid. examined the boy but found no
abnormality.
In adults with hypoglycemia, 50%
glucose solution is used for treatment of What is the most appropriate step?
severe hypoglycemia however in
children it is not recommended as it can a. Refer for the surgical consult
lead to death due to hyper osmolality. b. Reassure the mother
Oral sweeteners are given to a child c. Ask her to come back when the
with hypoglycemia if child is conscious bulge occurs
and cooperative. d. Admit to hospital
e. Request for an ultrasound
All other options are incorrect.
Answer detail:
48.A 5-year-old child with a history of The correct answer is a.
asthma is brought to your office by his
parents for advice. He had 2 episodes of The diagnosis is an inguinal hernia. It
asthma since last 6 weeks and he has follows the '6-2' rule:
been well in between the asthma For reducible hernia diagnosed between
episodes. The asthma symptoms respond births to 6 weeks, surgery is
to salbutamol inhaler. recommended within 2 days, 6 weeks to
6 months-surgery within 2 weeks and
What is the next step in his over 6 months surgery within 2 months.
management?
50.A 6-month-old male infant was brought
a. Fluticasone to your clinic due to characteristic cough
b. Sodium cromoglycate of pertussis.
c. Salmeterol
d. Oral Montelukast What is the best advice for the patient's
e. Keep using salbutamol as needed parents to reduce their risk of infection?

Answer detail: a. Commence a 3-dose pertussis


The correct answer is d. revaccination schedule
b. Immediate booster immunizations
This child is suffering from frequent for pertussis
intermittent asthma. Frequent c. A 10-day course of erythromycin
intermittent asthma is a class of asthma d. Arrange nasopharyngeal swabs
when the child gets more than one e. Immediate immunization with
episode of asthma in 6 weeks and no pertussis immunoglobulin
symptoms in between the flare-ups.
Answer detail:
Consider regular treatment with The correct answer is c.
Montelukast 4 mg once daily and review
response in 2-4 week. If symptoms do Pertussis is a respiratory infection with
not respond, consider regular treatment an incubation period of 7 to 20 days.
with a low dose of an inhaled
corticosteroid and review response in 4 In unvaccinated persons, B. pertussis is
weeks. highly infectious; spreading by aerosols
to 90% of susceptible household
contacts. Digoxin can enhance conduction
Natural infection does not provide long- through a bypass tract while slowing
term protection and repeat infection can down conduction through AV node.
occur. Ventricular fibrillation has been
reported in patients with WPW
Pertussis-containing vaccine is syndrome when treated with digoxin.
recommended in a 3-dose primary
schedule for infants at 2, 4 and 6 Verapamil can both shorten the
months of age. effective refractory period for accessory
All household contacts of patients with pathway and raise the risk of sudden
pertussis should receive erythromycin death in WPW patients should they
for 10 days to prevent further spread of develop atrial fibrillation.
the disease. So in this case, both digoxin and
verapamil are contraindicated.
Cotrimoxazole is recommended for
those intolerant of erythromycin. Losartan is angiotensin receptor blocker
and has no clinical use in SVT.
Since a primary vaccination course
requires three or more injections to Warfarin, an oral anticoagulant is also
protect against pertussis, infant not used in patients with SVT. It is used
vaccination cannot be effectively used in atrial fibrillation for prevention of
to protect unimmunized infants. thromboembolic events usually.

A booster dose of pertussis vaccine 52.Which of the following is correct about


should be considered for children aged combination of long acting beta 2 agonist
up to 8 years who has a contact with a with an inhaled corticosteroids in children
patient with pertussis however it’s with asthma?
effectively is questionable and
chemoprophylaxis should be considered a. Reduces severe exacerbations
for all cases. b. Increases severe exacerbations
Passive immunization with pertussis c. No effect on asthma overall
immunoglobulin is not effective in the d. Excellent for exercise induced
prevention of pertussis. asthma
e. Long acting beta2 agonists
51.What is the treatment of choice for an 8 decrease the need for inhaled steroids
year-old child who suffers from wolf white
Parkinson syndrome and supraventricular Answer detail:
tachycardia? The correct answer is b.

a. Digoxin Combination of long acting beta 2


b. Propranolol agonist with an inhaled corticosteroid
c. Verapamil significantly increases the risk of
d. Losartan asthma exacerbations in children.
e. Warfarin
The possible mechanism by which this
Answer detail: happens is thought that long acting beta
The correct answer is b. 2 agonists induces tolerance to short
acting beta2 agonists. This results in
Propranolol has replaced digoxin as the diminished response to child Norma
drug of choice for treatment of children rescue therapy with salbutamol.
with supraventricular tachycardia and
WPW.
Leukotriene receptor antagonists are Rectal diazepam is useful in the acute
superior to long acting beta 2 receptor management of seizures and is
agonists for step-up therapy in exercise indicated in following situations:
induced asthma.
1-A child with a pattern of prolonged
Long acting beta 2 agonists increase the seizures (usually longer than ten mins.)
need of inhaled corticosteroids in which have previously responded to
children and child is required to be on intravenous or rectal diazepam. 2-A
inhaled steroids continuously. child with clusters of repeated seizures
in whom oral treatment is inappropriate
53.A mother brings her 2-year old child with 3-A child with severe epilepsy who is
an episode of tonic-clonic seizure lasting remote from emergency services.
for one minute. The child also has a
history of fever with coryza for past three Rectal diazepam is not necessary for
days. children with mild or well-controlled
forms of epilepsy. The seizure, in this
How will you treat? case, has resolved in less than one
minute, so rectal diazepam is not
a. Oral paracetamol required.
b. Reassurance
c. EEG 54.Which of the following is an alarming sign
d. Sodium valproate in a four month old baby?
e. Rectal diazepam
a. Inability to follow activities with
Answer detail: eyes
The correct answer is b. b. Inability to roll over
c. Inability to sit without support
This child had a simple febrile seizure d. Child does not show pleasure when
which lasted for one minute. No seeing familiar people
treatment of seizure is required if it has e. Do not recognize parent
stopped itself.
Reassurance is necessary for simple Answer detail:
febrile convulsions. The onset of the The correct answer is a.
seizure. The convulsion may be a
terrifying experience for the parents to Following are the alarming signs in a
observe. four month old child when he/she:

Paracetamol has not been shown to 1. Does not make eye contact with
reduce the risk of further febrile people.
convulsions. 2. Does not turn to look for mother
An electroencephalogram (EEG) is not when she speaks
indicated for simple febrile seizure. 3. Is unhappy or unsettled most of the
time.
The rate of postictal EEG abnormalities
in children with complex febrile seizure By 5-7 months, a baby can roll from
is low, and studies that have shown an their back to their abdomen. At four
EEG abnormality have been unable to month-this is not an alarming sign. Also
translate this into a long term predictor child is unable to sit without support.
for future febrile seizure and afebrile This is also not an alarming sign.
seizures. At 6-9 months of age, it is concerning if
child does not show pleasure when
seeing familiar people and does not
recognize his/her parents. So these
findings are not concerning in a four
month old child. Answer detail:
The correct answer is b.
55.A mother brings her 4 months old male
baby with lump on the upper mid- This child has provisional diagnosis of
abdomen and crying. On examination it is hypertrophic pyloric stenosis. The
epigastria hernia. Pain is reproducible on investigation of choice in this situation
palpation. is abdominal ultrasound.

What will you advise her? Hypertrophic pyloric stenosis is more


common among males by a 4:1 ratio,
a. Surgical repair now particularly first born males. It occurs
b. Surgical repair after 6 months most often between 3 to 5 weeks of age
c. Conservative management until 4 and rarely after 12 weeks.
years of age
d. Reassure mother Symptoms can develop between 2 and 6
e. Prescribe paracetamol and review in weeks of Life. Projectile vomiting
4 weeks (without bile) occurs shortly after
eating. Until dehydration sets in, the
Answer detail: child feeds avidly and otherwise
The correct answer is a. appears well, unlike many of those with
vomiting caused by systemic illness.
This child has developed epigastria
hernia. Epigastria hernias are a type of Gastric peristaltic waves may be visible,
hernia that develop in the upper mid- crossing the epigastrium from left to
abdomen, between the sternum and the right. A discrete, 2- to 3-cm, firm,
umbilicus. movable, and olive-like pyloric mass is
sometimes palpable deep in the right
Epigastria hernia is unlikely to close side of the epigastrium.
naturally and is more likely to However these classic findings are not
incarcerate and cause pain by present in every case and diagnosis
strangulation. should still be suspected based on
history.
Indications of urgent surgical repair
include pain which is reproducible on Diagnosis is confirmed by abdominal
palpation of hernia. If a child presents ultrasonography showing increased
with epigastria hernia and without pain, thickness of the pylorus along with an
the surgical repair should be considered elongated pylorus. The classic
after the age of 6 months of age. electrolyte pattern of an infant with
pyloric stenosis is that of hypochloremic
56.A mother brings her 4-week-old child who metabolic alkalosis.
vomits after each feed. The child is
lethargic and hungry. On examination, no 57.A-12-year-old boy has BMI of 25. He is
epigastria mass is found. generally healthy and height is normal for
his age. According to BMI chart, where he
Which of the following investigation will stands in obesity classification?
confirm the diagnosis?
a. Obese
a. Barium meal b. Underweight
b. Ultrasound c. Overweight
c. Urine analysis d. Normal
d. Abdominal X-ray e. Need a dietician review
e. Stool culture
Answer detail: This girl has diagnosis of precocious
The correct answer is c. puberty and need investigations to find
out the cause.
Body mass index (BMI) is a measure of Investigations of suspected precocious
general adiposity. Body mass index is puberty in this girl include serum FSH,
defined as the individual's body mass LH and estradiol (females).
divided by the square of their height.
In young children, estimates of BMI Consider bone age x-ray (left wrist and
corrected for age percentiles are used elbow) and MRI if FSH and LH are
(see Obesity in children). increased. So this girl needs MRI scan of
BMI is classified as follows. the head to exclude any pituitary or any
1. Underweight with BMI less than 18.5. other brain tumor.
2. Healthy-18.5 to 24.9.
3. Overweight-25 to 29.9. 59.Which of the following is useful in the
4. Obese-30 or more management of moderate bronchiolitis?

So this child is overweight according to a. Nebulized Salbutamol


BMI classification. b. Oxygen
c. Inhaled hypertonic saline
58.A 6-year-old girl is accompanied by her d. Inhaled hypotonic saline
parents. They are worried about her as e. Oral steroids
she started developing her pubic hair,
breast and displays inappropriate sexual Answer detail:
behavior. The correct answer is b.

On examination, she height is more for The treatment of bronchiolitis mainly


her age and she has breast and pubic involved oxygenation and fluid therapy.
hair at Tanner stage 3.Serum FSH and LH In a child with moderate to severe
both are elevated. bronchiolitis, supplemental humidified
oxygen is useful. The oxygen should
What will do next? only be given to those children who are
hypoxic with oxygen saturation less
a. Reassure than 95%.
b. Follow up in 6 month
c. Serum FSH, LH There is no role of following treatments
d. MRI scan of brain choices in the management of
e. Monitor growth velocity bronchiolitis:

Answer detail: - Oral steroids.


The correct answer is d. - Salbutamol.
- Inhaled hypertonic saline.
Precocious puberty is characterized by - Inhaled hypotonic saline.
appearance of following features before - Cough suppressants.
the age of 8 years in girls:
Bronchiolitis is a viral illness caused by
- Development of secondary sexual a respiratory syncytial virus and it
characteristics. results in whee7e reduced oral intake
- Accelerated growth velocity. and respiratory distress in children
- Inappropriate sexual behavior. under two years of age.
- Mood disorders.
60.A 6-weeks old baby is brought to the
hospital with a history of gradually
worsening non-bilious vomiting over last
2 weeks. It has been becoming more What is most likely diagnosis?
forcible and the baby is eager to have his
formula feed after each vomiting. There is a. Neurogenic shock
no history of diarrhea. b. Hypovolemic shock
c. Obstructive shock
On examination, the capillary refill is 3 d. Cardiogenic shock
seconds. The baby has not gained any e. Inadequate fluid resuscitation
weight over last 2 weeks. There is no
jaundice. On abdominal examination, Answer detail:
there is an olive size mass palpable at The correct answer is a.
right upper quadrant when the baby is
supine. There is no rash. Rest of the This patient is most likely suffering from
examination is normal. neurogenic sock. Loss of sympathetic
lone prevents he expected tachycardia
Which of the following is the most response.
sensitive investigation to confirm the
diagnosis? The hallmarks neurogenic shock are
hypotension with either bradycardia or
a. CT abdomen normal heart rate despite fluid
b. Ultrasound abdomen resuscitation.
c. Urine microscopy
d. Barium swallow If the hypotension cannot be corrected
e. Laparotomy with fluids only, vasopressor therapy
may be required.
Answer detail:
The correct answer is b. As chest examination is clear, it is
unlikely obstructive shock (cardiac
This child has presented with clinical tamponade, pneumothorax) also it is
features of hypertrophic pyloric unlikely to be primary cardiogenic or
stenosis. Hypertrophic Pyloric Stenosis hypovolemic shock.
is due to progressive thickening of the
circular muscle of the pylorus. This Inadequate resuscitation is riot a
leads to gastric outlet narrowing. The correct option as information provided
condition usually presents between 2 indicated adequate fluid given.
and 6 weeks of age.
62.A 14-year-old boy is brought into the
Abdominal ultrasound is 95% sensitive emergency department with GCS of 8/15.
to diagnose the hypertrophic pyloric He has been unwell for last 24 hours at
stenosis. All other given investigations home with respiratory tract infection.
are not required except urine
microscopy which might help to rule out On assessment, his airway is intact and
urine tract infection. respiratory rate is 10 breaths per minute.
His oxygen saturation is 85% on room air
61.An 11-year-old male child is brought to and capillary refill is 3 with cold hands.
emergency following a fall into an unfilled His heart rate is 126 beats/minutes and
swimming pool and hit his head on the his blood pressure is 80/40 mmHg. His
concrete surface. Vital signs include heart capillary blood glucose less than 2
rate of 86/min, respiratory rate of 19/min mmol/l. He has been given intramuscular
and is hypotensive (50/30 mmHg) glucagon by the ambulance officer
despite adequate fluid resuscitation. CT however his blood sugar has not
scan of the head is normal. Chest improved.
examination is clear.
Urgent investigations shows sodium 130 Prostaglandin (PGE-1) is indicated in
mmol/l, potassium 5.5 mmol/l, venous cardiac lesions that depend on patent
blood glucose 1.6 mmol/l. ductus arteriosus to maintain adequate
pulmonary or systemic blood flow or to
Which of the following is the most likely promote adequate mixing.
diagnosis? These include:

a. Septic shock 1-Pulmonary atresia with intact


b. Neuroglycopenic symptoms due to ventricular septum.
hypoglycemia 2-Tricuspid atresia with intact
c. Addison's disease ventricular septum.
d. Hypovolemic shock 3-Critical coarctation of aorta.
e. Complicated insulin dependent 4-Hypoplastic left heart syndrome.
diabetes mellitus 5-Transposition of great arteries.

Answer detail: PGE-1 is not indicated in hyper-plastic


The correct answer is c. left ventricular syndrome, patent ductus
arteriosus, renal failure and heart
This patient has a typical presentation failure.
of acute adrenal Insufficiency. Clinical
findings in favor of this diagnosis 64.A 2-day old newborn is found to be
include hypotension, reduced CCS with irritable, crying all the time and sleeping
altered consciousness. very less. On examination he has
increased muscle tone, hyperactive deep
Laboratory findings include tendon reflexes and exaggerated Moro
hypoglycemia, hyperkalemia, reflex. Mother has been taking
hyponatremia and poor response to methadone daily until day of delivery.
glucagon injection. This is due to There is on electrolyte abnormality
cortisol and aldosterone deficiency. Any reported on routine investigations.
infection like upper respiratory tract
infection car unmask the symptoms. What is your next course of action?
This patient needs urgent fluid
resuscitation with steroid replacement a. Give naloxone
therapy. b. Give morphine
c. Start 5 percent dextrose infusion
In the presence of hyperkalemia, d. Give oral paracetamol
hypoglycemia and hyponatremia any e. Admit in intensive care unit for ten
other option are least likely to cause days
this type of presentation.
Answer detail:
63.Which of the following is the indication for The correct answer is b.
prostaglandin E-1 in a neonate?
This baby has developed neonatal
a. Transposition of great arteries abstinence syndrome.
b. Hyperplastic left heart syndrome
c. Patent ductus aretriosus Do not administer naloxone and
d. Renal failure naltrexone to babies of known or
e. Cardiac failure suspected opioid dependent women
during resuscitation or the neonatal
Answer detail: period. Use may precipitate severe rapid
The correct answer is a. onset of withdrawal associated with
seizures.
Withdrawal from heroin may he Toddler's diarrhea is suspected in a
clinically apparent within 24 hours from child with chronic diarrhea who is age 6
birth but is usually observed between months to 5 years, gaining weight and
24 and 72 hours. developing normally, and otherwise
healthy.
Withdrawal from methadone and
buprenorphine may be delayed until 3 - Children with toddler's diarrhea often
7 days after Birth, or beyond. have:

National Health and medical Research - 5-10 episodes of loose, watery, large
Council (NHMRC) recommends all opioid stools per day.
withdrawal should be treated with - Stools with undigested food particles.
morphine. - Diarrhea lasting weeks followed by
weeks of normal stools.
There is no evidence of hypoglycemia is
the baby on routine investigations. So it This child does not have symptoms
would be wise not to give any dextrose associated with dairy products and so
infusion. unlikely to suffer from lactose
intolerance.
Oral paracetamol is not helpful to treat
withdrawal symptoms. Other options a, b, d and e are incorrect.
This baby needs monitoring however rot
in intensive care for 10 days. General 66.A mother brings her 18-month old male
observation of withdrawal symptoms is child and is concerned as she believes,
required however baby should not be her child is not thriving well. On growth
discharged until day 7 after delivery. chart, the child is in the 25th percentile of
his weight since last 12 months.
65.A mother brings her 3-year old son with
history of loose stools containing peas What is the next best step?
and vegetables for last few days. The
child is gaining weight normally and is
otherwise healthy. Past medical history is a. Lactose free diet
insignificant. b. Sweat chloride test
c. Gluten free diet
What is the most likely diagnosis? d. Multivitamins
e. Reassure
a. Crohn's disease
b. Coeliac disease Answer detail:
c. Toddler's diarrhea The correct answer is e.
d. Lactose intolerance
e. Acute gastroenteritis Failure to thrive means the inability to
achieve the growth potential expected
Answer detail: for a child.
The correct answer is c.
Clinical features of failure to thrive
This child has clinical features include:
suggestive of Toddler's diarrhea.
It is also known as chronic nonspecific - The weight consistently below the 3rd
diarrhea of childhood. It is one of the to 5th percentile for age.
most common causes of chronic - Progressive decrease in weight to
diarrhea in otherwise healthy children. below the 3rd to the 5th percentile.
- A reduction in the percentile rank of 2 osteoporosis (bone weakening and risk
major growth parameters in a short of fragility fractures).
period.
Iron deficiency anemia is however most
Growth chart 'percentile' lines show the common finding amongst all of these.
reference range of weights and heights
for a particular age and gender. 68.A 9-year-old child was brought to the
emergency department with altered level
For example, 50 percent of the of consciousness and confusion. He has
population are expected to be below the history of type I diabetes mellitus and is
50th percentile. 25 percent below the on insulin. Capillary blood sugar is
25th percentile. 14mmol/L. What will you do next?

Half of all children at a given age are a. Chest X-ray


usually between the 25th and 75th b. Urine culture
percentiles. So reassure the mother that c. Arterial blood gases
child is normally thriving and no d. Blood culture
investigation needed at this stage. e. Give subcutaneous insulin

Parents and doctors should not feel Answer detail:


under any pressure to try that the The correct answer is c.
baby's weight should be on or near the
50th centile at any age. Diabetic ketoacidosis is the combination
of hyperglycemia, metabolic acidosis,
67.Absorption of which of the following is and ketonaemia.
most commonly impaired in untreated
coeliac disease? Following investigations arc needed
urgently to diagnose diabetic
a. Vitamin K ketoacidosis:
b. Iron
c. Vitamin B12 a. Capillary blood sugar level
d. Folate b. Venous or arterial blood gases
e. Calcium c. Blood or urinary ketone level
d. Other investigations like chest X-ray,
Answer detail: blood culture, urine culture, full blood
The correct answer is b. count etc.

Malabsorption in coeliac disease can All patients presenting with a blood


result in iron, folic acid and vitamin B12 glucose level more than 11.1mmo1/1
deficiency. should have blood ketones tested on a
But iron deficiency is seen more capillary sample.
frequently in patients with coeliac
disease than megaloblastic anemia due Urinalysis can be used for initial
to folic acid and vitamin B12 deficiency. assessment if blood ketone testing is
not available.
Absorption of fat-soluble vitamins A, D,
E, and K is also affected resulting in The biochemical criteria for DKA are:
their deficiency. a. Arterial pH less than 7.3 or
bicarbonate less than 15 mmol/l
Calcium and vitamin D malabsorption b. Presence of blood or urinary ketones
may cause osteopenia (decreased So in this patient is important to know
mineral content of the bone) or pH to fulfill the criteria for diagnosis of
diabetic ketoacidosis.
These antibodies cross react against
If ketones are negative, or the pH is kidney tissue. Proteinuria alone is very
normal in the presence of ketones, rare and if present, think of alternative
patients can be managed with diagnosis.
subcutaneous insulin. Streptococcal skin infection can cause
APGN if not treated with antibiotics.
69.A 9-year-old male child is brought in by
his mother with fever, headache and Seizures secondary to an acute
malaise for last three days. On hypertensive crisis are very rare.
examination, you notice facial puffiness
and peripheral edema around both 70.A mother brings a 10-year-old male child
ankles. Chest examination is significant to your clinic after a sports injury. She
for bilateral basal crackles. The rest of brought an avulsed tooth wrapped in a
the physical examination is normal. paper fold. On examination, you find the
maxillary central incisor is missing.
Blood pressure is 170/90. Heart rate is
90/min. Respiratory rate is 22/min What is the most appropriate next
Oxygen saturation is 98% on room air. management?

Dipstick urinalysis indicates blood, a. Re-implant the tooth under local


proteins. Hospital record shows that he anesthesia
was seen at an outpatient clinic about b. Immerse the tooth in the milk and
two weeks ago for a viral sore throat. referral to dentist for urgent review
c. Immerse the tooth in the milk and
Which of the following statement is true referral to dentist for review in next
regarding this condition? couple of days
d. Throw the tooth into medical waste
a. It is less common in Aboriginal and e. Reassure the mother that it will
Torres Strait Islanders grow back in few weeks
b. It is immune mediated condition
c. Proteinuria if alone is very Answer detail:
significant for diagnosis The correct answer is b.
d. It is unlikely to be caused by skin
infection This patient has presented with
e. Seizures secondary to acute completed disarticulation of the tooth.
hypertensive crisis is very common From its bony socket. This is called
avulsion of the permanent teeth.
Answer detail:
The correct answer is b. The clinical features of teeth avulsion
includes:
This child has developed clinical
features suggestive of acute post- - Pain.
streptococcal glomerulonephritis - Displacement of the tooth from the
(APGN). APGN is more commonly seen socket.
in Aboriginal and Torres Strait - Bleeding.
Islanders.
The fast frequently avulsed tooth in the
It is an immune mediated condition permanent dentition is the maxillary
caused by antibodies formed during central incisor, which predominantly
streptococcal throat and skin infection if presents in the 7-10 years' age group.
not treated with antibiotics.
The patient should he advised to
immerse the tooth in the milk and
should be seen by the dentist within 2 antibiotics will require admission to
hours ideally. Other storage options hospital.
include Harks' solution and contact lens
solution. 72.A 9-year-old boy has just been treated
for an anaphylactic reaction in a rural GP
Storage of the avulsed tooth in a clinic to which he responded well and is
compatible solution will prevent the stable now.
periodontal ligament from tying cut and
increases the possibility of successful What would be the next step in his
replantation. management?

The tooth should not be exposed to the a. Discharge home


dry air, and the water should be avoided b. Observe in GP clinic for one hour
as its osmotic effect causes cell death in and then send home
the periodontal ligament. c. Refer the child to nearest
emergency department
71.Which one of the following patients with d. Discharge home with Epipen
urinary tract infection will require e. Organize pediatric immunology
hospitalization? follow up

a. An infant less than 12-months Answer detail:


b. A 6-year old male child with family The correct answer is c.
history of urinary tract infection
c. A 5-year old female child with Any patient who has been treated for
family history of urinary tract infection anaphylaxis at home or in the
d. An infant less than 2-month of age community with an adrenaline auto
e. A 2-year old child with allergy to injector or in general practice should
penicillin then be sent to an emergency
department for post-acute treatment
Answer detail: observation.
The correct answer is d.
Current opinion is that a reasonable
As any infant younger than 2-month of minimum length of observation after
age is at increased risk of urosepsis or symptom resolution is 4-6 hours after
other serious concomitant infections the last dose of adrenaline.
and so requires treatment as an
inpatient. Organizing follow up with immunologist
is essential part of a first episode of
The family history of urinary tract anaphylaxis but it is next best step in
infection is significant only when there this situation.
is suspected urinary tract anatomical
abnormality suspected in a patient and 73.Which of the following is the risk factor
can be managed in the outpatient for the development of infant
department. hypertrophic pyloric stenosis in a 4 weeks
old infant?
Any allergy to penicillin is not an
indication of hospital admission. It a. Breastfeeding
requires giving the alternative oral b. Female
antibiotics as an outpatient. c. Asian background
d. Maternal smoking during pregnancy
The patients with UTI who are toxic, e. All of the above
dehydrated and unable to tolerate oral
Answer detail:
The correct answer is d. analysis shows it to be most likely a
viral meningitis.
The infant hypertrophic pyloric stenosis
is more common in maternal smoking Please click on the image attached to
during pregnancy. find out diagnostics of CSF analysis.

Hypertrophic Pyloric Stenosis due to 75.Which of the following is the most likely
progressive thickening of the circular to increase the risk of developmental
muscle of the pylorus. This leads to dysplasia of the hip?
gastric outlet narrowing. The condition
usually presents between 2 and 6 weeks a. Being male
of age. b. Breech presentation
c. Maternal smoking during pregnancy
The risk factor for the development of d. Asian origin
hypertrophic pyloric stenosis include: e. Iron deficiency

-Formula feeding rather than Answer detail:


breastfeeding The correct answer is b.
-Male
-Caucasian background rather than Developmental dysplasia of the hip is an
Asian background abnormal development of the hip joint.
-Firstborn The ball at the femoral head is not
-Maternal smoking during pregnancy stable within the acetabulum. The
-Positive family history ligaments of the hip joint which hold it
-Both erythromycin and azithromycin all together may also be stretched and
are associated with increased risk of loose. The hips dislocate after birth and
infant hypertrophic pyloric stenosis this may riot be noticed until your child
particularly when administered to starts to walk.
infants younger than 2 weeks.
The risk factors for the development of
74.An 8-year-old boy is brought to the developmental hip dysplasia include
emergency department with photophobia, being a girl, breech presentation and
irritability and neck stiffness. positive family history of hip dysplasia.

Lumbar puncture was performed and Being male, maternal smoking during
cerebrospinal fluid analysis show 50 pregnancy, being from Asian origin and
neutrophils, 100 lymphocytes, normal iron deficiency has no significant
proteins and normal glucose. association with the development of hip
dysplasia.
Which of the following is the most likely
diagnosis? Let is learn about breech presentation
as well.
a. Tuberculosis meningitis
b. Bacterial meningitis Complete breech: Here, the buttocks are
c. Viral meningitis pointing downward with the legs folded
d. Normal CSF findings at the knees and feet near the buttocks.
e. Encephalitis
Frank breech: In this position, the
Answer detail: baby's buttocks are aimed at the birth
The correct answer is c. canal with its legs sticking straight up in
front of his or her body and the feet
This patient has developed clinical near the head.
symptoms of meningitis and CSF
Footling breech: In this position, one or
both of the baby's feet point downward This child has Sp02 of 92% and fast
and will deliver before the rest of the breathing rate (normal respiratory rate
body. for 6-month baby is 50/min).

76.A 6-month-old child with Tetralogy of Main issues in management are parental
Fallout is brought to your clinic by his reassurance and education about
mother with a history of nasal discharge, minimal handling and frequent feeds.
low to high-grade fever, feeding
difficulties and wheezy cough since last I he diagnosis is usually limited to the
three days. Father of the child smokes first 72 months of life.
the cigarette a pack per day at home.
Steroids have not shown any benefit in
Physical examination is significant for the management of bronchiolitis and,
inspiratory crackles with an expiratory therefore, are not indicated.
wheeze. Respiratory rate is 60/min and
oxygen saturation is 92 % at room air. 77.You are working in a neonatal unit as an
intern. One of your patients developed
What is next best step in the hypoglycemia, and you have given
management of this patient? 2ml/kg of 10% dextrose.

a. Reassure the mother and educate Which one of the following is not
about minimal handling and frequent associated with neonatal hypoglycemia?
feeds
b. Admit this baby to hospital and a. Hepatomegaly
symptomatic care with supplemental b. Micropenis
oxygen, minimal handling and use of c. Mid-line defects
intravenous fluids d. Macrosomia
c. Start corticosteroid therapy e. Undescended testes
immediately
d. Start nebulized ribavirin and Answer detail:
monitor the response The correct answer is e.
e. 3 % inhaled normal saline reduces
the hospital stay and should be used Undescended testes are not associated
at first thing in hospital with syndromes causing hypoglycemia.

Answer detail: Hepatomegaly is associated with


The correct answer is b. glycogen storage diseases and fatty acid
oxidation disorders. It can cause
This patient has a clinical diagnosis of hypoglycemia.
acute viral bronchiolitis. The diagnosis
of bronchiolitis is clinical and bases on a Micropenis shows congenital
typical history of nasal discharge, fever gonadotropin deficiency and possible
and wheezy cough, and examination pituitary abnormalities. This may result
findings of inspiratory crackles and in cortisol deficiency. This may result in
expiratory wheeze. hypoglycemia.

Clinical indications for admission it Congenital pituitary deficiency can be


hospital in patients wile bronchiolitis associated with midline defects such as
include feeding difficulty, increased cleft lip, cleft palate, single central
work of breathing, increased respiratory incisor, and micro-ophthalmic.
rate and oxygen saturation less than 94
percent.
Hypoglycemia may be seen in these In the absence of palpable gonads, the
children as we I due to pituitary failure. most likely diagnosis is congenital
Macrosomia occurs in infants of diabetic adrenal hyperplasia secondary to 21
mothers and infants with severe hydroxylase deficiency.
congenital hyperinsulinism.
- 17-hytroxyprogesternne.
Hypoglycemia is seen very commonly in - Serum electrolytes.
neonates with a maternal history of - Plasma resin activity.
gestational diabetes and macrosomia.
79.A 6-year-old male child is observed by
78.A 4-week-old child is brought to your the school teacher to have sudden, brief
clinic by concerned parents. On lapses of consciousness without loss of
examination, baby has ambiguous postural control and no confusion after
genitalia. Gonads are palpable. Karyotype the episode. The teacher also observed
analysis confirmed 46xy genetic makeup. the child having rapid blinking of the
eyelids, chewing movements and clonic
Which of the following is the next movements of the hands during the
appropriate investigation? episodes.

a. X- ray abdomen The EEG is done after requesting the child


b. 17 Hydroxy-progesteron to hyperventilate. The result shows a
c. CT Scan pelvic region generalized, symmetric, 3-Hz spike-and-
d. Ultrasound of pelvic region wave discharge that begins and ends
e. Testosterone level suddenly, superimposed on a normal EEG
background.
Answer detail:
The correct answer is d. Which of the following is the most likely
diagnosis?
This patient has developed palpable
gonads with ambiguous genitalia. On a. Absence seizure
initial evaluation, a chromosomal b. Complex partial seizure
analysis should be performed. c. Simple partial seizure
d. Generalized tonic-clonic seizure
A pelvic ultrasound is performed as a e. Atypical absence seizure
next step to determine whether
mullerian structures (uterus) are Answer detail:
present. The correct answer is a.

A biochemical analysis is performed in The child has a typical presentation of


311 patients depending on palpability of absence seizure with typical EEG
gonads. findings.

1-Palpable gonads. Atypical absence seizure has features


like prolonged lapses of consciousness,
- Testosterone and dihydrotestosterone a more complex motor activity that may
(DHT) ratio. include focal or lateralizing features.
- LH and FSH.
- ACTH stimulation test. Atypical absence seizures are usually
- hCG stimulation test. associated with diffuse or multifocal
structural abnormalities of the brain
2-No palpable gonads: and, therefore, may accompany other
signs of neurological dysfunction such
as mental retardation.
nasal discharge, fever and wheezy
The atypical absence seizures are less cough, and examination findings show
responsive to anticonvulsants compared inspiratory crackles and/or expiratory
to typical absence seizures. wheeze.

Complex partial seizure shows focal In most of Europe and Australasia, the
seizure activity accompanied by a presence of inspiratory crackles is
transient impairment of the patients regarded as the more important
ability to maintain normal contact with examination finding, and the diagnosis
the surrounding environment. The is usually limited to the first 12 months
patient is usually unable to respond to of life.
visual or verbal commands during the
seizure and is accompanied by post- 82.A 12-year-old male presented with
iotal amnesia about the event. worsening of his asthma. He has past
medical history of asthma and recurrent
80.Which of the following organism is the chest infections.
most common cause of urinary tract
infections in children? Which ONE of the following if present
would indicate life threatening asthma?
a. Proteus mirabilis
b. E.coli a. Wheeze
c. Staph aureus b. Cyanosis
d. Enterobacter c. Speaks in sentences
e. Pseudomonas d. Peak expiratory flow more than
75%
Answer detail: e. Oxygen saturation 95%
The correct answer is b.
Answer detail:
Between 80 to 90% of urinary tract The correct answer is b.
infections ore caused by E.coli in
children. Cyanosis if present indicate life
threatening asthma episode. Wheeze is
Other organisms may include Klebsiella not a reliable indicator of severity of
pneumonia, Proteus mirabilis, asthma and it may be absent in severe
staphylococci species, pseudomonas asthma attack.
and enterobacters.
A patient with mild asthma can talk in
81.Which of the following is regarded as the sentences. Peak expiratory flow metre
most important clinical examination is usually more than 75% in mild
finding in a child with acute viral asthma In between 75-50%, it shows
bronchiolitis? moderate asthma and if below 50%
shows severe asthma.
a. Fever
b. Expiratory wheeze Oxygen saturation around 95% is sign
c. Nasal discharge of m Id asthma.
d. Inspiratory crackles
e. Cough with productive sputum 83.An 8-year-old boy is known to have
primary nocturnal enuresis. He is going to
Answer detail: attend a school camp next week and his
The correct answer is d. mother is worried about his night time
bed-wetting.
In a patient with acute viral
bronchiolitis, typical history includes What is the most appropriate treatment?
does not heal. Surgical repair is
a. Bed alarm therefore elective and not mandatory.
b. Nasal decompressing
c. Night-trainer The other types of perforation are not
d. Planned waking 'safe' and require specialist attention.
e. Any of the above
A continuously discharging central
Answer detail: perforation indicates granulation and a
The correct answer is b. risk of osteitis and bone destruction.
Marginal perforation carries the same
Decompression acetate is very useful risk.
for children with nocturnal enuresis who
are going for school camps. It can be A cholesteatoma is not a neoplasm but a
used oral or nasal each night. cystic lesion containing amorphous
Also the child should avoid water debris (and sometimes spicules of
loading before bed. cholesterol).

The bed alarm or night trainer is the It is formed through chronic infection
treatment of choice for primary and perforation of the eardrum with
nocturnal enuresis in all other ingrowth of squamous epithelium,
situations. forming a nest which becomes cystic.

Combination of decompression and the By progressive enlargement a


bed alarm can be used in cases with cholesteatoma can erode the ossicles,
persistent enuresis. labyrinth and adjacent bone and carries
the risk of cerebral abscess formation
Decompression is a synthetic analog of and meningitis.
arginine vasopressin. Decompression
works by decreasing urine volume at 85.An 18-month old female infant is brought
night and by decreasing intravascular to your clinic due to an onset of red
pressure. maculopapular rash on her trunk.
Symptoms reported by mother include
84.A 14-year old teenager is diagnosed with high grade fever and mild coryza for last
a tympanic membrane perforation 3 days however she has still been quite
secondary to chronic otitis media. Which active. Her temperature has returned to
of the following conditions will not normal today.
progress to significant complications if left
untreated? What is the most likely diagnosis?

a. Perforation associated with a a. Roseola Infantum


cholesteatoma b. Rubella
b. Marginal perforation with discharge c. Measles
c. Continuously discharging central d. Chicken pox
perforation e. Erythema Infectiosum
d. Perforation that is surrounded by
granulation tissue Answer detail:
e. Large dry central perforation The correct answer is a.

Answer detail: Roseala infantum is a viral infection


The correct answer is e. usually affecting children between the
ages of 6-18 months.
A dry central perforation will not
progress to complications, even if it
I he patient typically develops high d. X-ray
fever up to 40 degrees Celsius, but is e. Abnormal hip examination
otherwise not particularly unwell. There
may be mild cervical lymphadenopathy Answer detail:
and pharyngitis. The correct answer is d.

After three days, the temperature The x-ray of the hip is the investigation
usually returns suddenly to normal and of choice for screening developmental
the patient develops a red macular or dysplasia of the hip in beyond six
maculopapular non-desquamating rash months of age.
which is truncal, usually sparing the
'ace and limbs. The rash abates within All babies with risk factors for
Two days. developmental dysplasia of the hip
(Breech presentation, Oligohydramnios,
Rubella (German measles) is a viral Family history of DDH and underlying
infection typically characterized by rash, neuromuscular dysfunction) or with an
fever, and lymphadenopathy. The rash abnormal examination of the hip should
is usually an erythematous, discrete undergo a hip ultrasound and referral to
maculopapular exantherra that begins an orthopedic surgeon.
on the face and spreads caudally. It
usually disappears within three days but During first six months of age, the
may persist for eight days. ultrasound of the hip is the
investigation of choice.
The exanthema of measles is a
maculopapular, blanching rash 87.An 18-month-old male child is brought to
beginning or the face and spreading the emergency department by his mother
cephalocaudally and centrifugally to with generalized tonic-clonic seizure
involve the neck, upper trunk, lower lasting for 3 minutes an hour ago.
trunk, and extremities.
While you are assessing the child in the
The clinical manifestations of varicella emergency room, he again develops
(chicken pox) in healthy children tonic-clonic seizure which lasted for 5
generally develop within fifteen days minutes. The child is febrile and irritable
after the exposure and typically include with rectal temperature 39C.
a prodrome of fever, malaise, or
pharyngitis, loss of appetite, followed All immunizations are up to date. Rest of
by the development of a generalized the examination is unremarkable.
vesicular rash, usually within 24 hours.
What would you consider next?
The vesicular rash of varicella, which is
usually pruritic, appears in successive a. Full blood count
crops over several days. The patient b. Serum glucose
with varicella typically has lesions in c. Serum calcium
different stages of development on the d. No further investigation needed
face, trunk and extremities. e. Lumbar puncture

86.Which of the following is the best Answer detail:


screening test for developmental The correct answer is e.
dysplasia of the hip at 8 months of age?
88.A 4-year-old male child is brought by her
a. Ultrasound mother with a clean lacerated wound and
b. CT a history of 2 tetanus vaccinations.
c. MRI
What would you consider regarding his contraindication to pertussis
tetanus prophylaxis? vaccination.

a. DTPa vaccine 89.A 20-month-old baby is brought in by his


b. ADT and tetanus immunoglobulin parents with chronic diarrhea and weight
c. Tetanus toxoid and oral antibiotics loss. On examination, the baby has
d. Tetanus toxoid growth retardation, muscle wasting and
e. Tetanus immunoglobulin distended abdomen with hyperactive
bowel sounds. Chest examination is clear.
Answer detail: Laboratory investigations show decrease
The correct answer is a. serum iron or folate levels. Patient has
been on the lactose-free diet for 3 days
Vaccination against tetanus is included without resolution of symptoms.
in the National Immunization Program
schedule in Australia. What is the most likely diagnosis?
As a primary vaccine, tetanus toxoid is
given in combination with diphtheria a. Primary lactose intolerance
toxoid and acellular pertussis as DTPa b. Coeliac disease
vaccine. c. Cystic fibrosis
d. Bacterial over growth
Primary course of vaccination includes e. Chronic liver disease
DTPa vaccination at 2, 4 and 6 months
age. Then a first booster of DTPa is Answer detail:
given at 4 years of age. The correct answer is b.
Then a second booster of DTPa is given
between 12-17 years. So by the age of This patient has the clinical diagnosis of
17, an Australian young adult should coeliac disease. It presents as severe
have received 5 doses of tetanus toxoid malabsorction syndrome in children
containing vaccine. with chronic diarrhea, failure to thrive,
muscle wasting and hyperactive bowel
This child at the age of 4 year should be sounds.
ready to receive first booster of tetanus
toxoid containing vaccine. Primary lactose tolerance is due to
deficiency of lactase in a small intestine
However he has only received two and is a malabsorption syndrome
primary vaccinations for tetanus instead presenting with abdominal pain,
of three. So he needs to receive tetanus bloating, distension, flatulence and
toxoid containing vaccine in watery diarrhea. It usually resolves
combination with diphtheria toxoid and with a lactose-free diet for about 2
acellular pertussis. weeks.

As his wound is a clean laceration, no Cystic fibrosis is an inherited autosomal


immunoglobulin required and no recessive disease and is the most
antibiotics needed. He needs to have common cause of bronchiectasis and
either DTP (diphtheria tetanus and chronic suppurative lung disease in
pertussis) or DTPa which contains Caucasian children.
acellular pertussis. This will provide
additional immunity against diphtheria Bacterial overgrowth in the small bowel
and pertussis as well as tetanus which causes diarrhea and nutrient
is a primary concern here. malabsoption. Most patients with
bacterial overgrowth have intestinal
Diphtheria tetanus toxoid vaccine (ADT) stasis associated with underlying
should be used if child has allergy or anatomical abnormalities including
strictures, jejunal diverticulosis, motility treated with intravenous acyclovir
disorder and various operations that immediately.
create blind loops.
91.An 18-year-old mother brings her 5
Clinical manifestations of bacterial month old child in the emergency
growth include diarrhea, weight loss department. Mother is concerned that
and anemia due to vitamin B12 child has been crying excessively since
deficiency. yesterday and she noticed today that
child was unable to move the left arm.
90.A 4-year-old male child is brought to the There is no external bruising. Mother
emergency department by an ambulance does not remember any history of trauma
with 3 days history of fever, associated to the child. X-ray showed mid-shaft
altered consciousness. There has been fracture of the humerus.
progressive personality changes and
occasional drowsiness. What is the most appropriate next step?

An emergency lumbar puncture is done a. Social worker review t


which confirms the diagnosis of viral b. Admission to hospital
meningitis. c. Discharge and follow up with GP
d. Analgesia and external
Which of the following is the best choice manipulation in emergency
for this condition? department
e. Notify police
a. Ceftriaxone intravenously (IV)
b. Benzyl penicillin IV Answer detail:
c. Ciprofloxacin IV The correct answer is b.
d. Acyclovir IV
e. Cefuroxime IV This child has most likely diagnosis of
non-accidental injury to left humerus
Answer detail: which has resulted in fracture.
The correct answer is d.
In this case, there has been delay in
This case is suggestive of viral seeking advice (child crying since
encephalitis. Encephalitis is an yesterday) arc mother is denying any
inflammation of brain parenchyma.11 is trauma to the baby. A fracture is an
mainly caused by viruses although other evidence that physical trauma has
organisms including bacteria, happened.
mycoplasma, rickettsia and Histoplasma
can cause encephalitis. So the most appropriate things to do
include admission to the hospital (do
Suspect it when a viral prodrome is not discharge this Patient hone as risk
followed by irrational behavior, altered of further injuries is high), orthopedics
consciousness and possibly cranial review and notification to child
nerve lesions. protection services.

In this case, the cerebrospinal fluid Child protection services can be


analysis reveals viral meningitis thus contacted any time throughout
viral etiology is entertained. Australia.

This patient should be hospitalized and Notification of suspected child abuse is


treatment will be supportive. Suspected mandatory and later on, if no child
herpes simplex encephalitis should be abuse is proven, health practitioners are
protected by Australia National. Law Cystic fibrosis presents with frequent
from any legal action by parents. respiratory and gastrointestinal
infections from the birth. This child was
Social worker review and notification to previously healthy, so cystic fibrosis is
police not required. Any external unlikely diagnosis.
manipulation in children must be guided
by orthopedic team. 93.Which of the following statement is
correct regarding anti-glomerular
92.A mother brings her 3 year-old male basement membrane disease?
Aboriginal child for passing loose stools
over last 2 weeks. The child lost 2 kg a. Antibodies are directed against non-
weight after the onset of diarrhea. Child collagenous domain of type IV
looks fatigued. Clinical examination is collagen
remarkable for mild dehydration. b. Antibodies are directed against
collagenous domain of type IV
What is the most likely diagnosis? collagen
c. Type IV collagen is found in skeletal
a. Lactose intolerance muscle
b. Food poisoning d. It is most commonly seen in people
c. Coeliac disease above 50
d. Cystic fibrosis e. If treated with steroids, relapses
e. Giardiasis are very common

Answer detail: Answer detail:


The correct answer is e. The correct answer is a.

This child has signs and symptoms Anti-glomerular basement membrane


consistent with diagnosis of Giardiasis. disease is also known as Goodpasture's
It is caused by a parasite called as disease.
Giardia lamblia.
It is caused by the development of auto
Symptoms of Giardiasis include: antibodies against non-collagenous
domain of type IV collagen.
1. Diarrhea, which may be watery,
usually lasting more than 1-2 weeks. Type IV collagen is also found in the
lung and hemoptysis may also be a
2. Weight loss. feature, particularly in smokers.

3. Frequent loose or pale, greasy stools Young males are most commonly
which may float in the toilet bowl. affected, but it can occur at any age or
in any gender.
4. Fatigue.
Treatment options include plasma
Lactose intolerance may develop in 20- exchange, corticosteroids and cytotoxic
40% of cases of giardiasis and last therapy.
several weeks. However it is not
primary diagnosis here. It treatment is stared early, the disease
can be cured and relapses are rare.
Coeliac disease presents titer
introduction of glutin-containing solid Patients present with macroscopic
diet in a child during weaning. It starts hematuria progressing rapidly to
during first year of life. oliguria and renal failure it left
untreated.
This woman has developed postnatal
94.Which ONE of the following is unlikely to depression. The diagnosis of postpartum
require ant-D administration in an Rh depression should be considered in
negative woman who have not actively woman after child birth with following
formed their own Anti-D? clinical features persisting more than 2
weeks:
a. Threatened abortion before 12
weeks of gestation - Low mood, tearfulness, anxiety
b. Spontaneous abortion - Insomnia
c. External cephalic version - Loss of appetite
d. Significant closed intra-abdominal - Loss of weight
trauma - Loss of self-esteem
e. Termination of pregnancy - Loss of enjoyment
- Impairment of functions in normal life
Answer detail: - Poor memory and concentration.
The correct answer is a.
Postnatal depression occurs in 10-30%
There is insufficient evidence to suggest of women in first 6-12 months
that a threatened miscarriage before 12 postpartum.
weeks gestation requires Anti-D.
Postnatal blues occur in 80% of the
Indications to use ant-Din all Rh women during first 2 weeks after child
negative women who have not actively birth. Patient feels sad or depressed
formed their own Anti-D include: with mood swings and irritability.
Insomnia, tiredness and lack of
- Spontaneous abortion confidence are also commonly seen in
- External cephalic version postpartum blues. This is transient
- Significant closed intra-abdominal condition and lasts for 4 to 14 days
trauma only.
- Termination of pregnancy.
- Chorionic Villus Sampling. All other options are incorrect.
- Ectopic pregnancy.
- Threatened abortion after 12 weeks of 96.Which of the following is an absolute
gestation. contra-indication to use progestogen-only
pills?
95.A mother brings her 4 months old son as
he wakes her a loran night She is tearful, a. Rifampicin
crying and anxious. She tells you that she b. History of liver disease
has not slept since he was born. She has c. Migraine
lost her appetite and does not enjoy her d. Malabsorption syndrome
life. e. Smoking

What is the most likely diagnosis? Answer detail:


The correct answer is a.
a. Insomnia
b. Postpartum psychosis Concomitant use of enzyme-inducing
c. Sleep deprivation drugs results in decreased efficacy of
d. Postpartum depression progestogen-only pills and can result in
e. Postpartum blues pregnancy.

Answer detail: Enzyme inducers include anti-epileptic


The correct answer is d. drugs, rifampicin and barbiturates etc.
Enzyme inducing drugs (rifampicin) are c. One preterm birth before 34 weeks
an absolute contraindication to use d. Carcinoma cervix
progestogen-only pills. e. Vaginal polyp

As rifampicin and rifabutin are Answer detail:


particularly potent enzyme inducers, The correct answer is a.
women using these drugs long term,
should be advised to switch to a method A minority of recurrent second trimester
that is unaffected by enzyme-inducing abortions are due to cervical
drugs. insufficiency exclusively.

History of liver disease and Cervical cerclage is a preventive


malabsorption syndrome are relative measure and is performed at 12 to 14
contraindications to use progestogen- weeks of gestation.
only pills.
Indications of cervical cerclage include:
Progestogen-only pills are suitable for
patients with a history of smoking and - Two or more consecutive prior second
migraine. trimester pregnancy losses or
- Three or more preterm births before
97.Warfarin is contraindicated during 34 weeks of gestation
pregnancy. Which of the following
complications develop if warfarin is used Short cervix less than 24 weeks of
in the second trimester of gestation? gestation without any preterm birth is
not a reason to do cerclage.
a. Foetal chondrodysplasia Punctata Similarly carcinoma cervix and vaginal
b. Hypercalcaemia polyp are also not indications to
c. Facial anomalies perform cerclage.
d. Maternal cerebral bleeding
e. Foetal optic atrophy 99.A 35-year-old lady presented with
irritability, anxiety, breast tenderness and
Answer detail: headaches about 5 days before the onset
The correct answer is e. of menses every month, since the last 6
months.
Warfarin is contraindicated in the first
trimester as it causes foetal These symptoms are relieved with the
chondrodysplasia punclata. If used in onset of menses. She has to take a week
second and third trimester, warfarin off from work due to these symptoms.
causes foetal optic atrophy and mental
retardation. She is non-alcoholic and does not use any
regular medication.
Warfarin does not cause hypercalcaemia
and foetal facial anomalies if used Which of the following is the
during second and third trimester.
a. Premenstrual dysphonic disorder
98.Which of the following is the indication to b. Premenstrual syndrome
perform cervical cordage at 14 weeks of c. Normal menstrual physiology
gestation? d. Generalized anxiety disorder
e. Depression
a. 2 or more consecutive prior second
trimester pregnancy losses Answer detail:
b. Short cervix at <24 weeks with no The correct answer is b.
prior preterm birth
This patient fulfils the criteria for the She has been having normal regular
diagnosis of the premenstrual periods every 4 weeks. She has done a
syndrome. home pregnancy test which was positive.
On examination, there is no tenderness
Diagnostic criteria for premenstrual and the cervical os is closed. The pelvic
syndrome include affective and somatic ultrasound shows empty sac and there is
symptoms during the 5 days before no fluid in the pouch of Douglas.
menses in each of the 3 prior menstrual
cycles. What is the most likely diagnosis?

Affective symptoms include: a. Complete abortion


b. Missed abortion
- Depression c. False positive pregnancy test
- Anger outbursts. d. Ectopic pregnancy
- Irritability e. Septic abortion
- Anxiety
- Confusion Answer detail:
- Social withdrawal. The correct answer is a.
Somatic symptoms include breast
tenderness, abdominal bloating, This patient has developed clinical
headache and swelling of extremities. features of early pregnancy loss. A
complete miscarriage is a miscarriage in
Symptoms are usually relieved within 4 which early pregnancy tissue is
days of onset of menses and are present completely expelled.
in the absence of any pharmacological
therapy, or drug or alcohol use. - Gestation typically 6 to 8 weeks
- No fetal activity ever detected
The patient suffers from identifiable - Empty sac enlarge sac with minimal
dysfunction in social or economic structures without fetal heart activity
performance. - Beta hCG levels show an initial rise
and then fall.
Premenstrual dysphonic disorder is a
severe form of premenstrual syndrome A missed miscarriage is the one with
and is characterized by a severe feeling ultrasound features consistent with a
of sadness, emotional liability with non-viable or non-continuing
frequent tearfulness, loss of interest in pregnancy, even in the absence of
daily activities, decreased clinical features. Early pregnancy tissue
concentration, fatigue, insomnia and may be partially expelled.
feeling of being overwhelmed or out of
control. An incomplete miscarriage is a
miscarriage in which early pregnancy
Symptoms must have been there for tissue is partially expelled.
most of the during the last 12 months
and have resulted in interference with The clinical features of ectopic
daily activities. pregnancy include lower abdominal
pain, significant vaginal bleeding, and
Generalized anxiety disorder and amenorrhea. This patient has
depression alone are the unlikely intrauterine gestational sac (although
diagnoses. empty) and there is no fluid in Pouch of
Douglas. So this is unlikely ectopic
100. A 17-year-old girl comes to pregnancy.
your office with a history of vaginal
bleeding after 6 weeks of amenorrhea.
101. A 27-year-old female c. Primary infertility of greater than
presents with sudden onset of a severe three months
headache at 37 weeks of gestation. She d. Symptoms of endometriosis
also feels dizzy and nauseated. Her unresponsive to initial treatment
mother noted slurred speech and e. Primay infertility of greater than
repetitions of words while she was one year
complaining of the headache. She is
clinically stable and has not developed Answer detail:
any obstetric complication. She has no The correct answer is c.
history of a migraine.
Australian health system works well
What will you do next in her coordinated between specialists in large
management? hospitals and general practitioners at
surgeries and clinics. It is always
a. Aspirin recommended to refer the patient if
b. CT brain specialist opinion is needed.
c. MIZI brain
d. Lumbar puncture You should refers patient to
e. Observation in the ward Gynecologists or Obstetrician if you find
a patient with:
Answer detail:
The correct answer is b. • Unexplained pelvic pain.
• Pelvic mass which is tender on bi-
This patient has developed clinical manual vaginal examination
features of subarachnoid hemorrhage • Primary infertility of greater than a
and needs CT scan of the brain to rule year
out intracranial hemorrhage. • Patient with suspected diagnosis of
With proper shielding of the uterus, endometriosis unresponsive to initial
radiation exposure to the fetus is treatment.
minimal.
Primary infertility of greater than three
Aspirin should be avoided until CT scan months is not an indication of referral to
of the brain is performed and any the specialist.
intracranial hemorrhage is ruled out.
103. A 30-year-old woman
Lumbar puncture should be carried out developed deep vein thrombosis at 30
if the patient has clinical features week of pregnancy. She was treated with
suggestive of subarachnoid hemorrhage low molecular weight heparin. Now she is
and CT head is normal. It should be at 34 weeks of pregnancy and her
performed 12 hours after the onset of a expected date of delivery is in next 4
headache. weeks. What would be your advice for
today?
Observation in the ward is not
appropriate without ruling out a. Low molecular weight heparin
subarachnoid hemorrhage. should be switched to unfractionated
heparin
102. You are principal house b. Continue low molecular weight
officer at a GP clinic. You would refer all heparin until delivery
of the following presentations for c. Switch to warfarin and check INO in
specialist opinion except? 2 days
a. Unexplained persistent pelvic pain d. Use both low molecular weight
b. A pelvic mass tender on bimanual heparin and warfarin for 5 days
vaginal examination e. Cease all anti-coagulants
Answer detail:
The correct answer is a.

A patient with deep vein thrombosis


during pregnancy requires 3-6 months
treatment with low molecular weight
heparin (LMWH) or unfractionated
heparin.

LMWH may be associated with an


increased risk of epidural hematoma in
women receiving an epidural anesthetic
in labor.

Four weeks prior to anticipated delivery,


LMWH should be switched to
unfractionated heparin.

You might also like